vendredi 4 octobre 2019

Message à l'attention de VB et penseur saumon

Pour une fois je vais demander de l'aide aux deux commentateurs cités (évidemment si d'autres s'en sentent le courage ils sont les bienvenus) dans le titre car à mon avis ils ont des connaissances suffisantes pour répondre à la question posée par Josh dans le fil des commentaires de mon billet intitulé L'effondrement ne serait pas inéluctable, ouf on respire ! ; voici les éléments nécessaires à la compréhension de cette question (allez au boulot les p'tits gars !) et tout d'abord la question à laquelle il faut répondre pour satisfaire Josh :
le point clé est la figure 2.1 du premier rapport du GIEC qui montre que (dans la fenêtre d'absorption du CO2 à 15 microns), très peu de rayonnement parvient à la tropopause, en contradiction avec le mécanisme décrit par M. Dufresne, et en bon accord avec le mécanisme que j'ai patiemment élaboré après des mois de recherche; voir:

https://www.ipcc.ch/site/assets/uploads/2018/03/ipcc_far_wg_I_full_report.pdf

Mais pour bien comprendre d'où cette question provient, voici le (long) fil de commentaires duquel j'ai essayé d'expurger tout ce qui était inutile et prenait de la place, bonne lecture !

Josh27 août 2019 à 10:25

Deux théories s’opposent ; je cite le GIEC (Glossaire du rapport AR5) :
Effet de serre : Effet radiatif de tous les constituants de l’atmosphère qui absorbent le rayonnement infrarouge. Les gaz à effet de serre, les nuages et, dans une moindre mesure, les aérosols absorbent le rayonnement terrestre émis à la surface de la Terre et dans l’atmosphère. Ces constituants émettent un rayonnement infrarouge dans toutes les directions…

Pas du tout disent les climato-sceptiques, les molécules des gaz à effet de serre sont désactivées par collision avec les molécules de l’air environnant avant d’avoir eu le temps d’émettre un rayonnement infrarouge.

Qui a raison ? Remarquons tout d’abord qu’il s’agit d’un problème d’interaction entre un rayonnement électromagnétique et la matière, donc d’un problème de pure physique impliquant le rayonnement du corps noir (Planck), la théorie cinétique des gaz (Boltzmann), la théorie du rayonnement thermique
(Kircchoff) et la thermodynamique (Carnot).

Il ne s’agit donc en aucun cas d’un problème climatique, et les climatologues ne sont sans doute pas les mieux placés pour en parler ; notons par exemple l’article du météorologue J.L. Dufresne « L’ effet de serre atmosphérique : plus subtil qu'on ne le croit ! » dans lequel l’auteur considère que l’atmosphère rayonne comme un corps noir alors que tout physicien sait qu’à pression et température ambiantes, un gaz rayonne selon la loi de Kirchhoff.

Le Professeur Geuskens est donc tout à fait dans son domaine de compétence pour traiter ce sujet, et manifestement il touche sa bille en la matière ; grâce à la loi de Maxwell-Boltzmann, il calcule la fréquence des collisions entre molécules, soit plus d’un milliard par seconde ; il cite ensuite l’article de B.M.Smirnov, J. Phys. D: Appl. Phys. 51, 214004 (2018) qui donne un temps de 0, 62 secondes pour la désactivation des molécules de CO2 par absorption du rayonnement infrarouge.

Donc, d’après Geuskens, il n’y a pas photo : les molécules de CO2 n’ont pas le temps de se désactiver par rayonnement, elles se désactivent par collision avec les molécules environnantes ; tout ça tient la route, ce mécanisme me paraît le plus vraisemblable, c’est celui que j’ai appris à l’école, et il est injuste de le balayer d’un revers de main sous prétexte que Geuskens est vieux et Belge.

Maintenant, je me pose la question : sur quels calculs, quelles publications le GIEC se base-t-il pour affirmer que la désactivation des molécules de CO2 se fait par radiation ? pas une ligne dans les milliers de page des différents rapports, pas une ligne sur internet (j’ai sans doute mal cherché).

Donc, plutôt que de s’envoyer des insultes qui ne font pas avancer le schmilblick, il faudrait que les tenants du mécanisme décrit par le GIEC donnent leurs propres estimations chiffrées pour ces deux paramètres : temps de désactivation du CO2 et temps entre deux collisions , afin de pouvoir choisir en toute objectivité entre les deux mécanismes..

Pourquoi est-ce si important de décider quel est le mécanisme qui prévaut ?
parce que si la désactivation par radiation est confirmée, le CO2 n’absorberait pas le rayonnement infrarouge émis par la terre (donc ne serait pas un gaz à effet de serre ?) ; en revanche, si c’est la désactivation par collision qui est confirmée, alors tout l’édifice du GIEC s’écroule.

Tout le monde (y compris le GIEC au paragraphe 2.2.2 de son premier rapport en 1990) admet que l’absorption du rayonnement infrarouge dans les fréquences d’absorption du CO2 est déjà totale ; un ajout de CO2 dans l’atmosphère ne pourrait donc rien absorber de plus puisque tout est déjà absorbé ; en d’autres termes, un ajout de CO2 n’aurait aucune influence sur le réchauffement climatique ; rien que ça.
Je passe sur mes réponses à Josh qui ne sont pas intéressantes dans le cas présent, je me contente de reprendre les commentaires de Josh qui sont importants pour suivre le fil de ses pensées :

Josh27 août 2019 à 16:58

[…] je n'ai pas trouvé sur quoi se base le GIEC pour affirmer que la désexcitation des molécules de CO2 se fait par radiation; je ne vais pas demander ça à ceux qui pense qu'elle se fait par conduction, je connais déjà leur réponse; je voulais seulement connaître la réponse des tenants de la théorie du GIEC, que je n'ai trouvé nulle part, et j'ai pensé que vous pourriez peut-être me la donner.
Josh15 septembre 2019 à 11:48

[…] La théorie du GIEC est résumée dans le Glossaire de son cinquième rapport; je cite:

"Effet de serre: Effet radiatif de tous les constituants de l’atmosphère qui
absorbent le rayonnement infrarouge. Les gaz à effet de serre, les nuages et,
dans une moindre mesure, les aérosols absorbent le rayonnement terrestre
émis à la surface de la Terre et dans l’atmosphère. Ces constituants émettent un
rayonnement infrarouge dans toutes les directions, mais, toutes choses étant
égales par ailleurs, la quantité nette de rayonnement émis vers l’espace est
alors inférieure que ce qu’elle aurait pu être en l’absence de ces constituants,
compte tenu de la baisse de la température avec l’altitude dans la troposphère
et de l’affaiblissement de l’émission qui en découle. L’augmentation de
la concentration de gaz à effet de serre accroît cet effet; on fait parfois
référence à cette différence en utilisant l’expression effet de serre additionnel.
L’augmentation de la concentration des gaz à effet de serre découlant
d’émissions anthropiques se traduit par un forçage radiatif instantané. La
surface terrestre et la troposphère se réchauffent en réponse à ce forçage,
rétablissant graduellement l’équilibre radiatif au sommet de l’atmosphère."

Il y a au moins quatre modèles différents expliquant l'effet de serre: Arrhenius (1ère version), Angström, Arrhenius (2ème version) et Ekholm; le modèle cité par le GIEC est celui d'Ekholm; il a été élaboré il y a cent ans, à l'époque où il y avait peu ou pas de spectrométrie infrarouge et encore moins de satellite; ce modéle est expliqué dans l'article de J.L. Dufresne:

https://www.hprevot.fr/plus-subtil.pdf.

Cet article contient au moins deux erreurs flagrantes qui confirment que ce modèle n'est pas valable; de fait les mesures satellites ne confirment pas l'élévation de température au sommet de l'atmosphère prévue par ce modèle.

Le réchauffement climatique n'est pas contesté; il reste à cerner le mécanisme par lequel le CO2 y contribue pour pouvoir préciser son impact sur ce réchauffement.

Josh17 septembre 2019 à 12:59

[…]

Sur le fond maintenant : tout ce que j’ai écrit est déjà publié, ce n’est pas la peine que je le fasse, ça ne ferait qu’ajouter à la cacophonie ambiante ; je me suis donc contenté de lire les articles existants publiés par les deux bords, et c’est assez facile, avec un peu de connaissances scientifiques, de voir qui raconte des carabistouilles ou qui est compétent ; on trouve de tout, du meilleur au pire, et rassurez-vous, autant des deux côtés de la ligne de démarcation.

Il n’y a pas de consensus actuel émanant de la communauté scientifique ; les rapports du GIEC sont critiqués, parfois à juste titre, parfois non ; pour ce qui concerne le modèle de l’effet de serre, le GIEC a eu raison de choisir le modèle d’Ekholm, où l’effet de serre se produit au sommet de l’atmosphère ; certains climato-sceptiques l’ont compris et ne le contestent pas, au contraire, mais la plupart ne l’ont pas compris.

Quant à moi, il m’a fallu plus de deux mois pour vraiment comprendre le mécanisme complet par lequel le CO2 influe sur le réchauffement climatique, et je n’ai trouvé aucun article clair sur ce sujet.

Il reste un fait qui est admis par les deux parties, sceptiques ou non : l’absorption du rayonnement infrarouge par le CO2 est saturé ; ceci a été mis en évidence par Angström, et n’est pas contesté (voir par exemple le paragraphe 2.2.2 du premier rapport du GIEC).

Cela veut dire qu’au-dessus d’environ 200 ppm de CO2, l'absorption du rayonnement infrarouge n’augmente pratiquement plus ; on peut doubler la teneur en CO2, voire plus, l’absorption n’augmente que très peu ; je n’ai trouvé nulle part l’explication à ce mystérieux phénomène que personne pourtant ne conteste.

La conséquence inévitable est que l’augmentation actuelle du taux de CO2 dans l’atmosphère n’a qu’un effet marginal sur le réchauffement climatique ; vous posez une très bonne question : si ce n’est pas le CO2, alors qu’est-ce que c’est ? il y a de nombreuses réponses possibles; la dernière que j’ai lu explique que c’est corrélé à la vitesse du soleil par rapport au centre de gravité du système solaire ( ?) ; de toutes façons, j’ai résolu le problème qui m’intéressait, et cet autre problème ne m’intéresse pas.
[…]

Josh21 septembre 2019 à 11:03

[…] voici le lien vers le premier rapport du GIEC:

https://www.ipcc.ch/site/assets/uploads/2018/03/ipcc_far_wg_I_chapter_02.pdf

Je cite un extrait:

"For carbon dioxide, as has alreadybeen mentioned, parts of the spectrum are already so opaque that additional molecules of carbon dioxide are even less effective."

Pourquoi ceci n'est pas rappelé dans le cinquième rapport (ni dans aucun des autres rapports d'ailleurs) ? Je vous laisse y réfléchir.

Je répondrai plus tard en détail sur la quasi-saturation de l'absorption du rayonnement infra-rouge par le CO2, car c'est bien évidemment la clef de toute l'histoire.

Josh27 août 2019 à 10:32

Maintenant, je vais vous dire le fond de ma pensée ; dans son premier rapport, le GIEC montre un graphique où l’on voit que très peu de rayonnement est transmis à la tropopause dans la bande des fréquences d’absorption du CO2

Cela veut bien dire que la plupart du rayonnement est absorbé, et non pas réémis ; mais comme déjà expliqué ci-dessus, la conséquence imparable est qu’un ajout de CO2 ne pourrait rien absorber de plus puisque tout est déjà absorbé ; c’est sans doute pour contourner cette objection que le GIEC a inventé sa théorie du forçage radiatif.

Je ne sais pas quel crédit il faut apporter à cette histoire qui dit que Thatcher était en conflit avec les mineurs Anglais, qu’elle a voulu promouvoir l’énergie nucléaire en discréditant le CO2, et que le GIEC a été créé pour ça.

Quoi qu’il en soit, ce mécanisme de désactivation par rayonnement semble peu probable ; de même, le fait que la tropopause se réchauffe n’a pas été confirmé par les observations ; enfin, imaginer que la tropopause à -19°C puisse réchauffer la basse atmosphère à 14°C est contraire au deuxième principe de la thermodynamique : un corps froid ne peut pas céder de chaleur à un corps plus chaud que lui.

Devant tant d’objections , les tenants du mécanisme décrit par le GIEC devraient, autrement que par des insultes, justifier leurs affirmations, et démontrer, calculs à l’appui, que les molécules de CO2 se désactivent par radiation et non par collision.

C’est la seule façon de clouer le bec aux climato-sceptiques, mais on en est loin aujourd’hui.

Felix qui potuit rerum conoscere causas.

Josh27 août 2019 à 15:08

Les molécules de CO2 excitées par le rayonnement infrarouge se désactivent-elles par radiation ou par conduction ?

Josh22 septembre 2019 à 11:04

Table 2.2 page 52: la variation de l'absorption du CO2 est donnée par la formule:

Delta F = 6.3 ln (C/Co)

Prenons un exemple: pour une concentration de 25 ppm, l'absorption serait de 10 W.m-2.

Pour 50 ppm de CO2, l'augmentation de l'absorption (Delta F) serait de:

6.3 ln (50/25) = 4,5 W.m-2, donc l'absorption serait de 10 + 4.5 = 14,5 W . m-2, en augmentation de 45 %.

Pour 75 ppm, même calcul: 6.3 ln (75/25) = 7,1 W.m-2, soit un total de 17,1 W.m-2, en augmentation de 18 % par rapport à la valeur précédente.

Pour 375 ppm, 6.3 ln (375/25) = 17,6 W.m-2, donc un total de 27,1 W.m-2 en augmentation de 1,6 % par rapport à la valeur calculée pour 350 ppm, et pour 400 ppm
6.3 ln (400 / 25) = 18, soit un total d'absorption de 10 + 18= 28 W.m-2, en augmentation de 1,5 % par rapport à la valeur précédente.

Donc, les 25 premiers ppm ont causé une augmentation de 45 % de l'absorption du CO2, les 25 suivants une augmentation de 18 %; l'augmentation de l'absorption décroit régulièrement suivant une loi logarithmique et n'est plus que de 1,6 % quand la concentration augmente de 350 à 375 ppm, puis de 1,5 % quand elle augmente de 375 ppm à 400 ppm.

C'est ce qu'on appelle la saturation de l'absorption du CO2; ce phénomène est confirmé par les mesures expérimentales qui sont bien expliquées dans l'article de Dufresne:

https://www.hprevot.fr/plus-subtil.pdf

Dufresne ne peut pas être suspecté d'être sceptique puisque son article explique et justifie le fonctionnement du modèle d'effet de serre choisi par le GIEC, et il travaille à l'Institut Simon Laplace dont le directeur est J. Jouzel, membre du GIEC.

En retenant la formule logarithmique citée plus haut, le GIEC accepte de facto la saturation de l'absorption du CO2; il n'y a plus guère de débat la-dessus, et chacun comprend que les 2 ppm de CO2 que nous rajoutons tous les ans dans l'atmosphère ne peuvent influencer le climat que de façon marginale.

La bagarre s'est maintenant déplacée sur le front des rétro-actions; le GIEC affirme que ces 2 ppm génère des actions beaucoup plus importantes que la seule absorption du rayonnement infra-roude, par rétro actions positives sur la mer, les nuages, etc...

Bien entendu, les sceptiques disent que s'il y a rétro-action, elles sont négatives; il s'agit des deux côtés de calculs très spéculatifs, invérifiables par l'expérience, et chacun peut raconter ce qu'il veut; est-ce encore de la science ?

A ce stade-là j'ai demandé son avis à François-Marie Bréon qui m'a répondu textuellement ceci :
Je réponds rapidement car je suis débordé.

Tout d’abord, la formule donné par ton contradicteur, Delta F = 6.3 ln (C/Co), est une approximation basée sur des calculs plus complexes qui font intervenir le profil de température de l’atmosphère, la présence de nuage, la vapeur d’eau. J’insiste sur le fait que ce n’est pas qqchose qui est codé dans les modèles de climat, mais qui est déduit de calculs plus complexes.

Ensuite, cette approximation a été calculée sur des simulations entre 170 et quelques centaines de ppm. Elle ne s’applique pas sur des valeurs très faibles (25 ppm) jamais observées sur les derniers millions d’années.

Prenons donc un cas réaliste : On est passé de 270 ppm à 410 ppm aujourd’hui. Le Delta_F est donc de 6.3 Ln(410/270) = 2,6 W/m2

Aujourd’hui, on augmente de 2-3 ppm/an. Donc, sur 30 ans, on va se prendre au moins 70 ppm. Donc un forçage supplémentaire de 6.3 Ln (480/410)= 1 W.m2
Ce n’est pas rien. 1 W.m2 n’est PAS marginal
Donc non, il n’y a pas « saturation » même si il est vrai que les ppm supplémentaires comptent moins que les ppm passés.
Je transmets cette réponse à Josh qui enchaine :
Josh23 septembre 2019 à 10:47

Merci pour votre réponse; c'est vraiment sympa d'avoir consulté une personne compétente; je vous ai dit qu'avec un peu de bagage scientifique on pouvait facilement voir si son interlocuteur raconte des carabistouilles; ce Monsieur connait manifestement son sujet, et je suis d'accord avec ce qu'il a écrit.

En dérivant la loi de Boltzmann, (je sais, c'est compliqué mais finalement pas tant que ça), on a:

(Delta T) = T/4 x (Delta F) / F

où T est la température de la terre, soit 288 °K, Delta F vaut 1 W.m-2, et F est la radiation totale de la terre, soit 238,5 W.m-2; donc l'augmentation de température correspondant au supplément d'absorption par les 70 ppm de CO2 supplémentaires émis dans les trente prochaines années sera:

288/4 x 1 / 238,5 = 0,3 °C.

Le CO2 à lui tout seul ne peut pas justifier une augmentation significative de température; c'est pourquoi le GIEC a fait appel aux rétroactions positives.

C'est vrai que quand la température augmente, l'évaporation augmente, et ça augmente l'effet de serre de la vapeur d'eau, mais ça fait aussi des nuages qui réduisent l'ensoleillement; de toutes façons, je ne veux pas m'aventurer sur ce terrain...

Encore merci pour vos réponses, à vous et à M. Bréon.

Josh23 septembre 2019 à 16:37

[…] pour moi, séparer les aspects naturels et anthropiques du réchauffement, c'est un peu comme séparer le jaune du blanc dans l'omelette , mais faisons confiance aux spécialistes; j'ai donc lu le chapitre auquel M. Bréon a participé, où j'ai relevé cette phrase:

"La contribution des forçages naturels se situe probablement entre −0,1 °C et 0,1 °C, et celle de la variabilité naturelle interne probablement entre −0,1 °C et 0,1 °C"

Donc, on a tenu compte de la variabilité naturelle du climat, mais celle-ci est nulle en moyenne; le climat a toujours varié, pourquoi se serait-il arrêté de le faire ? pas étonnant que les sceptiques râlent.

Pour ce qui et des rétroactions, sur la vapeur d'eau, l'albédo et les nuages, est-ce qu'on les attribue au CO2 qui, on l'a vu ne génère que 0,3 °C de hausse sur les 30 prochaines années, ou à une des autres causes possibles ?
[…] 

Josh27 septembre 2019 à 10:12

[…] j’ai lu la controverse entre MM. Bréon et Gervais ; dans sa réponse, M. Bréon cite l’article :

Dufresne, JL et J Treiner, L’effet de serre atmosphérique : plus subtil qu’on ne le croit ! ; La Météorologie, 72, Fev 2011.

Je vois que M. Bréon et M. Dufresne collaborent tous deux à l’Institut Pierre-Simon Laplace ; or cet article de M. Dufresne me pose deux problèmes ; je ne sais pas dans quelle mesure vous pouvez m’aider à les résoudre, mais je vous expose quand même mon point de vue :

1) M. Dufresne explique que les molécules de CO2 excitées par le rayonnement infrarouge émis par la terre se désactivent par effet radiatif dans toutes les directions ; petit à petit, ce rayonnement progresse en altitude, couche après couche, pour finalement arriver à une altitude d’émission où l’air est assez raréfié pour ne plus absorber ce rayonnement qui est donc envoyé dans l’espace.

Quand la concentration en CO2 augmente, l’altitude d’émission augmente aussi ; la température baisse avec l’altitude, et l’émission du CO2 décroit avec la température ; il y a moins de rayonnement envoyé dans l’espace, donc la température de l’atmosphère ce réchauffe.

2) De nombreuses sources concordantes considèrent au contraire qu’à température et pression ordinaires dans la basse troposphère, l’agitation thermique provoque plus d’un milliard de collisions par seconde ; le temps de désactivation par rayonnement de la molécule de CO2 varie suivant son état d’excitation, mais serait toujours supérieur à un millième de seconde.

Donc, les molécules de CO2 se désactivent principalement par collision avec les molécules de l’air environnant, en augmentant leur énergie cinétique.

Selon ce mécanisme, la totalité du rayonnement émis par la terre dans les bandes d’absorption du CO2 est absorbée dans les 100 premiers mètres de l’atmosphère, et aucun photon ne parvient plus haut .

La totalité du rayonnement étant absorbée, les ajouts de CO2 ne peuvent causer aucune absorption supplémentaire, donc n’ont aucun effet sur le réchauffement ; beaucoup de sceptiques s’arrêtent à cette conclusion. 
Mais certains auteurs vont plus loin ; ils expliquent que tout corps solide, liquide ou gazeux émet un rayonnement (sauf au zéro absolu) ; même à la tropopause, où il fait -50°C et où on a l’impression que tout est gelé, en fait la température est +223 K, suffisamment chaud pour créer une agitation thermique significative.

Du fait de cet agitation thermique, les molécules de CO2 sont excitées par collision avec les molécules de l’air environnant ; à basse altitude, comme on l’a déjà vu, la désexcitation se fait par collision avec d’autres molécules ; mais l’air se raréfie avec l’altitude, et donc les collisions entre molécules deviennent de moins en moins fréquentes ; à partir d’une certaine altitude, l’effet radiatif prend le pas sur les collisions ; ce rayonnement est envoyé dans l’espace, ce qui refroidit la terre.

Lorsque la concentration en CO2 augmente, une partie de ce rayonnement est absorbée, ce qui réchauffe l’atmosphère ; ce serait donc le rayonnement émis par l’atmosphère qui générerait l’effet de serre, et pas celui émis par la terre comme expliqué dans l’article de M. Dufresne.

On peut aussi penser que l’altitude d’émission augmentant, la température baisse et le rayonnement diminue, comme expliqué par M. Dufresne, et c’est là le deuxième problème.

3) Certains auteurs expliquent qu’à l’altitude de la tropopause, l’analyse infrarouge de l’atmosphère montre que le CO2 est toujours 100 % absorbant, au moins dans sa bande principale à 15 microns ; si c’est vrai, l’effet radiatif ne prend le pas sur les collisions que dans la stratosphère, où la température augmente avec l’altitude.

Avec l’augmentation de la concentration en CO2, on aurait donc deux phénomènes qui se compensent au moins partiellement: une augmentation du rayonnement due à l’augmentation d’altitude, et une diminution due à l’absorption.

J’espère que j’ai été assez clair ; bien sûr, Planck, Boltzmann, Kirchhoff et Carnot pourraient m’aider à résoudre ces problèmes, mais comme ils sont morts, il faudrait trouver quelqu’un de compétent et d’impartial pour le faire ; connaissez-vous quelqu’un compétent et impartial ?

Josh2 octobre 2019 à 11:33

[…] J'ai écrit à M. Bréon pour lui expliquer mon problème concernant l'article de M. Dufresne; d'un côté je suis raisonnablement certain que j'ai raison, d'un autre côté je n'arrive pas à croire que des gens aussi compétents puissent faire de telles erreurs, et je voudrais avoir leur opinion sur mon texte.

Je n'ai pas reçu de réponse à ce jour; peut-être que mon e-mail est allé dans le spam; vous semblez avoir accès direct à M. Bréon, serait-ce abuser de vous demander de lui transmettre mon texte (ci-dessous).
[…]

 Josh cite alors textuellement son message à François-Marie Bréon :
Josh2 octobre 2019 à 11:39

J’ai lu votre controverse entre MM. Bréon et Gervais ; dans votre réponse, vous citez l’article :

Dufresne, JL et J Treiner, L’effet de serre atmosphérique : plus subtil qu’on ne le croit ! ; La Météorologie, 72, Fev 2011.

Je vois que M. Dufresne et vous collaborez tous deux à l’Institut Pierre-Simon Laplace ; or cet article de M. Dufresne me pose deux problèmes ; je ne sais pas dans quelle mesure vous pouvez m’aider à les résoudre, mais je vous expose quand même mon point de vue :

1) M. Dufresne explique que les molécules de CO2 excitées par le rayonnement infrarouge émis par la terre se désactivent par effet radiatif dans toutes les directions ; petit à petit, ce rayonnement progresse en altitude, couche après couche, pour finalement arriver à une altitude d’émission où l’air est assez raréfié pour ne plus absorber ce rayonnement qui est donc envoyé dans l’espace.

Quand la concentration en CO2 augmente, l’altitude d’émission augmente aussi ; la température baisse avec l’altitude, et l’émission du CO2 décroit avec la température ; il y a moins de rayonnement envoyé dans l’espace, donc la température de l’atmosphère ce réchauffe.

2) Le résultat de la résolution des équations de la théorie cinétique des gaz montre, selon plusieurs sources concordantes , qu’à température et pression ordinaires dans la basse troposphère, l’agitation thermique provoque plus d’un milliard de collisions par seconde ; le temps de désactivation par rayonnement de la molécule de CO2 varie suivant son état d’excitation, mais serait presque toujours supérieur à un millième de seconde.

Donc, les molécules de CO2 se désactivent principalement par collision avec les molécules de l’air environnant, en augmentant leur énergie cinétique.

Selon ce mécanisme, la totalité du rayonnement émis par la terre dans les bandes d’absorption du CO2 est absorbée dans les 100 premiers mètres de l’atmosphère, et aucun photon ne parvient plus haut ; ce résultat est confirmé par la figure 2.1 du premier rapport du GIEC : à la tropopause, l’absorption autour de la fréquence de 15 µm est quasi-totale.

La totalité du rayonnement émis par la terre étant absorbée, les ajouts ultérieurs de CO2 ne peuvent causer aucune absorption supplémentaire, et n’ont donc aucun effet sur le réchauffement.

3) Beaucoup de sceptiques s’arrêtent à cette conclusion., mais certains auteurs vont plus loin ; ils expliquent que tout corps solide, liquide ou gazeux émet un rayonnement (sauf au zéro absolu) ; même à la tropopause, où il fait -50°C et où on a l’impression que tout est gelé, en fait la température est +223 K, suffisamment chaude pour créer une agitation thermique significative.

Du fait de cet agitation thermique, les molécules de CO2 sont excitées par collision avec les molécules de l’air environnant ; à basse altitude, comme on l’a déjà vu, la désexcitation se fait par collision avec d’autres molécules ; mais l’air se raréfie avec l’altitude, et donc les collisions entre molécules deviennent de moins en moins fréquentes ; à partir d’une certaine altitude, l’effet radiatif prend le pas sur les collisions ; ce rayonnement est envoyé dans l’espace, ce qui refroidit la terre

Lorsque la concentration en CO2 augmente, une partie de ce rayonnement est absorbée, ce qui réchauffe l’atmosphère ; ce serait donc le rayonnement émis par l’atmosphère qui générerait l’effet de serre, et pas celui émis par la terre comme expliqué dans l’article de M. Dufresne.

On peut aussi penser que l’altitude d’émission augmentant, la température baisse et le rayonnement diminue, comme expliqué par M. Dufresne, et c’est là le deuxième problème.
4) Toujours selon la figure 2.1 ci-dessus, l’analyse infrarouge de l’atmosphère montre que le CO2 est toujours 100 % absorbant à l’altitude de la tropopause,, au moins dans sa bande principale à 15 microns ; si c’est vrai, l’effet radiatif ne prend le pas sur les collisions que dans la stratosphère, où la température augmente avec l’altitude.

Avec l’augmentation de la concentration en CO2, on aurait donc deux phénomènes qui se compensent au moins partiellement: une augmentation du rayonnement due à l’augmentation d’altitude, et une diminution due à l’absorption.

Je pense qu’on ne peut pas prévoir quelle sera la température en 2050 ou en 2100 si on n’a pas une idée claire du mécanisme selon lequel le CO2 agit sur le réchauffement ; une fois que ce mécanisme est accepté, il faudrait effectuer les calculs correspondants, et vérifier que le résultat de ces calculs concorde avec les observations satellites.

Les formules empiriques du genre ΔF = 6.3 ln (C/Co) sont tout à fait insuffisantes compte tenu de l’importance du problème.

 Ce message à Bréon reprend plusieurs points du commentaire posté le 27 septembre à 10h12.

Josh2 octobre 2019 à 17:37

[…] j'ai un problème qui est de savoir par quel mécanisme le CO2 influe sur le climat (avant prise en compte d'éventuelles rétroactions); c'est précisément ce qu'explique l'article de M. Dufresne, qui est abondamment critiqué par les sceptiques; il me parait essentiel et équitable d'avoir aussi l'opinion de M. Bréon sur ces critiques pour que je puisse me faire ma propre idée; je ne sais pas si je lui répondrai que je suis raisonnablement certain qu'il a tort, et même si je le faisais, je ne vois rien de mal à ça; après tout, Einstein et Bohr non plus n'étaient pas d'accord...

J'ai donc redemandé à François-Marie Bréon ce qu'il en pensait, voici sa réponse :
[…] je n’ai pas bien compris après deux lectures, et j’ai donc choisi de ne pas répondre.

Tu peux éventuellement réagir à sa dernière phrase

Les formules empiriques du genre ΔF = 6.3 ln (C/Co) sont tout à fait insuffisantes compte tenu de l’importance du problème.


En disant que les formules empiriques sont DEDUITES des modèles de climat, mais que toutes les simulations climatiques sont faites avec un calcul explicite des forcages radiatif (ie calcul des flux infrarouges dans l’atmosphère en tenant compte de la composition atmosphérique et de la présence de nuages) et n’utilisent donc jamais cette formule

Réponse transmise à Josh qui continue :
Josh2 octobre 2019 à 17:14

Merci d'avoir essayé; je lis les articles des deux côtés (sceptiques ou pas) et j'essaye de voir qui dit juste; dans ce cas précis, je suis raisonnablement certain que M. Dufresne se trompe, mais ça parait tellement incroyable que je voulais avoir l'opinion de M. Bréon; je suis aussi raisonnablement convaincu qu'on ne peut pas construire un modèle prédisant la température en 2050 si on n'est pas d'accord sur les mécanismes par lesquels le CO2 influe sur le climat avant (et c'est essentiel) de prendre en compte les éventuelles rétroactions; après toutes ces recherches, j'ai mon opinion que j'ai expliquée dans mon texte, mais cela m'aurait beaucoup aidé de connaitre celle de M. Bréon.
[…]

Josh3 octobre 2019 à 11:58

J'ai vu que M. Bréon est Normalien; quand on est à l'ENSCP (l'Ecole où j'ai fait mes études à Paris), il suffit de traverser la rue d'Ulm pour entrer à l'ENS; M. Bréon et moi, on a appris à peu près les mêmes choses (même si le concours d'entrée à Normale Sup est plus relevé); après l'Ecole, je me suis orienté vers l'Industrie et lui s'est orienté vers la Recherche; aujourd'hui, il est logiquement devenu beaucoup plus compétent que moi sur le climat, et c'est pourquoi je sollicite son expertise.

Je ne crois pas tout savoir, bien au contraire je suis tout à fait disposé à apprendre des spécialistes de la question.

Dans mon texte, je décris deux mécanismes possibles d'action du CO2 sur le climat (avant, et c'est essentiel, avant prise en compte d'éventuelles rétroactions); ces deux mécanismes sont incompatibles; lequel est conforme aux lois de la physique ? et pourquoi ?
[…]

Josh4 octobre 2019 à 11:29

Bonjour; comme indiqué précédemment, je cherche à obtenir un avis contradictoire sur le texte que vous avez déjà soumis à M. Bréon; je comprends bien qu'il a bien d'autres choses à faire que de me répondre, et je ne lui en veut pas.

Vous mentionnez deux personnes ayant des connaissances scientifiques, VB et penseur saumon; puis-je vous demander de leur communiquer ce texte, en leur demandant leur avis ? le point clé est la figure 2.1 du premier rapport du GIEC qui montre que (dans la fenêtre d'absorption du CO2 à 15 microns), très peu de rayonnement parvient à la tropopause, en contradiction avec le mécanisme décrit par M. Dufresne, et en bon accord avec le mécanisme que j'ai patiemment élaboré après des mois de recherche; voir:

https://www.ipcc.ch/site/assets/uploads/2018/03/ipcc_far_wg_I_full_report.pdf

La controverse scientifique a toujours existé, même au plus haut niveau (c'est pour cela que j'ai cité Einstein et Bohr) et elle fait progresser la science; j'aimerais recevoir des critiques constructives sur mon texte afin de le corriger ou de le compléter.

D'avance merci pour ce que vous pourrez faire.

Et moi aussi je dis merci d'avance à VB et penseur saumon, ainsi qu'à tout autre intervenant ayant quelque chose à dire d'intelligent sur le sujet, pour ce que vous pourrez faire afin d'éclairer la lanterne de Josh !



127 commentaires:

  1. Bien que n'étant pas nommé , je me permet d'intervenir.
    C'est une présentation interessante et troublante faite par Josh.
    Il est important d'apporter des réponses claires et argumentées á ce type de questionnement.
    Je suis un peu troublé par une des réponses de Mr Bréon. Lorsqu'il dit que la formule de forcage "empirique" en fonction de la concentration de CO2 est issue des modéles .
    Si c'est une formule approximée á partir des modéles , ce n'est donc pas "empirique".
    https://www.linternaute.fr/dictionnaire/fr/definition/empirique/

    Je ne pense pas qu'il y ait d'experience ou des moyens de mesures permettant de confirmer cette formule. Est ce le cas ?
    J'ai cru lire que l'on était pas capable de mesurer le bilan radiatif de la Terre . Est ce exact ?

    Sinon sur le fait que le CO2 que l'on ajoute a de moins en moins d'effet, je ne pense pas que cela soit une "Breaking News" . C'est effectivement clairement montré avec cette formule déduite des modeles du GIEC (loi log), mais également sous entendu dans le fait que l'ECS soit considerée comme une constante. (Imaginons que ECS = 3 degC on est parti de 300 ppm , on arrive á +3 deg pour 600 ppm et á +6 degC pour 1200 ppm)


    Du coup, je me pose des questions á propos des "tipping points" . Si je comprend bien "un tipping point" introduit une non-linéarité aggravant la situation. Je n'ai pas trouvé de passages satisfaisants dans le rapport du GIEC á ce sujet .

    RépondreSupprimer
    Réponses
    1. « Bien que n'étant pas nommé , je me permet d'intervenir. »

      J'ai bien écrit « évidemment si d'autres s'en sentent le courage ils sont les bienvenus », donc vous êtes le bienvenu ; si je ne vous ai pas mentionné dans mon appel à l'aide c'est que je n'avais pas compris que vous aviez des qualifications scientifiques du même ordre que celles de VB et de penseur saumon, il serait peut-être bien de préciser quelles études vous avez faites et éventuellement quelle est la nature de votre travail, cela pourrait nous éclairer;)

      La définition d'« empirique » est la suivante selon le Larousse : « Qui s'attache exclusivement à l'observation et au classement des données sans l'intervention d'un système ou d'une théorie a priori. / Qui se conduit en tenant compte essentiellement des circonstances, des opportunités, sans faire référence à des principes arrêtés ; pragmatique. » Dans ce cadre-là il me semble que ce qui sort d'un modèle est parfaitement empirique, puisque c'est ce qu'on observe et que ce n'est en aucune façon une théorie, donc à mon humble avis Bréon a raison sur ce point.

      Pour le reste je me garderai d'intervenir et laisse la discussion libre d'accès aux personnes compétentes pour traiter la question de Josh.

      Supprimer
  2. ben non j'ai pas vraiment de compétences scientifiques (BTS Tourisme)... J'essaye juste d'utiliser mon cerveau . ;-)

    RépondreSupprimer
    Réponses
    1. Alors non, désolé BenHur mais vous ne pouvez pas participer utilement à la discussion qui est bien trop technique pour vous et moi, veuillez laisser la parole à des gens qui ont des compétences qui nous font défaut.

      Moi aussi j'essaye d'utiliser mon cerveau, mais justement mon cerveau à un moment me dit stop, et je l'écoute.

      Supprimer
  3. Mais on peut etre discuter tous les deux ? Non .
    Je me posais une question en lisant ce billet troublant .
    Il est dit par le GIEC que la variabilité naturelle est entre +/- 0.1 degC . Ok
    Le GIEC dit aussi que le facteur aggravant l'effet de serre du CO2 sont les retroactions positives. Je me demandais alors : Imaginons qu'il n y ait pas le forcage anthropique . Prenons avant 1850 .
    Le climat varie librement entre [-0.1; + 0.1 degC] . Mais comment revient on á l'equilibre si les feedbacks sont positifs. On devrait diverger .... Je sais que ma question est bete , mais quand meme ... Je ne sais pas si mon questionnement est clair.

    RépondreSupprimer
    Réponses
    1. « Mais on peut etre discuter tous les deux ? Non . »

      NON, pas sur des points qu'on ne comprend pas.

      Si j'ai posté ce billet c'est pour avoir l'avis de spécialistes, pas des opinions de gens sympathiques qui n'y connaissent pas grand chose, et ça vaut pour moi bien sûr.

      Supprimer
  4. Une première réflexion au sujet de la variabilité naturelle. J'ai découvert grâce à votre blog et je vous en remercie qu'il n'y a plus aucun réchauffement en Suisse l'hiver depuis 30 ans. La conclusion logique est que si on admet l'hypothèse d'un réchauffement du au CO2 la variabilité naturelle peut se révéler tout aussi importante (pour complétement annuler ce réchauffement l'hiver). Conclusion qui devrait logiquement en amener d'autres mais restons-en là.
    La théorie de Geuskens - qui est partagée par nombre de physiciens - va totalement à l'encontre du GIEC. J'ai longtemps cherché et je cherche encore, mais personne n'arrive à lui donner tort ce qui veut dire qu'au bout d'un moment il va falloir lui donner raison! Nous avons pourtant les outils pour mesurer le rayonnement infrarouge émit par le CO2 mais nous ne mesurons rien, pourquoi?? Ce qui au passage réglerait définitivement la question.
    Cordialement.
    Hugo.

    RépondreSupprimer
    Réponses
    1. Cher Hugo va falloir vous abstenir, ce que je dis à BenHur vaut également encore plus pour vous.

      Supprimer
    2. Cher Hugo ,
      Je pense que vous confondez réchauffement climatique et modification du pattern meteo regional.

      Jamais le GIEC n'a dit que toute les temperatures au cours de l'année devaient augmenter en moyenne de X degC en tous points du globe. Il y a hétérogénité (et c'est bien normal puisqu'il y a des climats regionaux trés différents).
      Il est absolument reconnu par le GIEC que le rechauffement climatique :
      1) n'est pas egal en toute region du globe
      2) modifie les pattern meteo regionaux . ( et d'apres ce que j'ai compris c'est un des grands axes de travail actuel)

      D'autre part vous vous limitez également á un effet saisonnier en ne parlant que de l'hiver
      Donc dans le cas de la Suisse, le rechauffement climatique a vraisemblablement modifié légérement son pattern meteo et donne des hiver assez stables alors que les étés la temperature elle augmente.

      Supprimer
    3. « J'ai découvert grâce à votre blog et je vous en remercie qu'il n'y a plus aucun réchauffement en Suisse l'hiver depuis 30 ans. »

      Justement, dans l'article auquel vous faites allusion (https://sogeco31.blogspot.com/2019/04/le-deni-est-un-vilain-defaut.html) VB intervenait avec ceci : « Tendance statistiquement non significative, comme je l'ai déjà signalé à plusieurs reprises. Dans un signal bruité (et plus on restreint le champ des observations - un lieu, une région, une saison - plus le signal risque d'être bruité), la variabilité naturelle, ou des facteurs locaux (changement de vents dominants, fonte de la neige, ... par exemple) peuvent inverser la tendance, souvent temporairement. »

      C'est bien pour cela que j'aimerais avoir l'avis de VB sur la question de Josh qui ne concerne pas la météo en Suisse, donc merci de ne pas divaguer en dehors du sujet de l'article.

      Supprimer
  5. Désolé Ged ... Je ne suis pas non plus un scientifique spécialisé en climatologie .... Mais si ça peut aider (en attendant que ceux que tu interpelle se manifestent ;) ) :

    Ce que je mentionnais déjà dans mes premiers articles sur le sujet

    [- CLIMATO-SCEPTIQUES : CHEVALIERS DE L'ORDRE DE LA TERRE PLATE ?
    (http://2013-continuum.blogspot.fr/2010/12/climato-sceptiques-chevaliers-de-lordre.html)

    - LES CHEVALIERS DE L'ORDRE DE LA TERRE PLATE...
    (http://2013-continuum.blogspot.fr/2011/01/la-suite-du-precedent-article-hari-est.html)]

    ==> L'augmentation présumée de la température a été prédite bien avant d'être détectable dans l'atmosphère, en fait bien avant de savoir que le CO2 était vraiment en train d'augmenter.
    Ceci a été prédit pour la première fois par Arrhenius en 1896 à partir d'idées extrêmement simples sur l'équilibre radiatif, puis fut reproduit en utilisant de la physique moderne des rayonnements par Manabe et collaborateurs dans les années 60.
    Aucune de ces prédictions ne reposait sur des modèles de circulation générale, qui sont apparus dans les décennies suivantes et ont permis des prévisions plus détaillées.
    Mais la prédiction de base du réchauffement est fondée sur des principes de physique vraiment fondamentaux ayant trait à l'absorption des infrarouges par les gaz à effet de serre, à la théorie radiative du corps noir, et à la thermodynamique de l'atmosphère saturée.
    Chacun de ces éléments a été vérifié avec une très bonne précision par des expériences de laboratoire et des observations de terrain.

    Pendant un temps, une incertitude persistait sur le fait de savoir si la vapeur d'eau allait amplifier le réchauffement avec l'amplitude simulée par les premiers modèles d'équilibre radiatif, mais une ou deux décennies de travail supplémentaire à la fois d'observation et de théorie ont montré qu'il n'y a pas vraiment de raison de douter du calcul de cette rétroaction par les modèles de circulation générale.
    Modifiée en introduisant l'effet refroidissant des aérosols anthropiques, la théorie rend compte de façon satisfaisante de l'allure de la variation de température des 20e et 21e siècles.

    Aucune autre théorie basée sur des principes physiques quantifiables n'a pu faire de même. Si quelqu'un arrivait avec la brillante idée que, disons, le réchauffement global est dû à la pluie de Phlogistique tombant de la Lune, cela n'effacerait pas d'un coup tout ce que l'on sait sur la thermodynamique, sur l'absorption des infrarouges, l'équilibre radiatif, et la température.
    Au contraire, c'est le boulot de l'avocat du Phlogistique de quantifier les effets du Phlogistique sur l'équilibre radiatif, et de les ajouter de manière cohérente aux forçages climatiques existants.
    Quasiment toutes les tentatives d'enfoncer la théorie de l'effet de serre anthropique ont perdu de vue ce principe simple mais incontournable !...

    Cf.
    http://www.realclimate.org/index.php/archives/2007/11/les-chevaliers-de-lordre-de-la-terre-plate-part-i-allgre-and-courtillot/langswitch_lang/fr/

    Et :
    https://forums.infoclimat.fr/f/topic/18970-conf%C3%A9rence-de-vincent-courtillot-lundi-8-juin/

    RépondreSupprimer
    Réponses
    1. Merci Daneel mais je pense que cela ne répond pas à la question très technique de Josh qui pointe une soi-disant discordance de vues entre Bréon et Dufresne.

      Supprimer
  6. oui bien sur BenHur, mais il n'est question dans mon propos que d'une seule région qui se réchauffe effectivement le reste de l'année et au final annuellement. L'effet radiatif - s'il existe - n'est en aucun cas saisonnier.
    Hugo.

    RépondreSupprimer
    Réponses
    1. Hugo, ce sont des phrases comme « si on admet l'hypothèse d'un réchauffement du au CO2 » ou « L'effet radiatif - s'il existe » qui risquent de vous valoir de vous retrouver à la poubelle si vous persistez à venir inoculer ici votre totale méconnaissance en climatologie, comme en science d'ailleurs j'imagine ; il serait au passage intéressant de connaitre votre cursus scolaire et universitaire, juste pour voir.

      Supprimer
  7. Un DESS en génie physique et je suis à ce titre très impatient de découvrir l'argumentaire très scientifique de vos commentateurs réfutant Geuskens et prouvant que l'effet radiatif du CO2 dans notre basse atmosphère n'est pas un mythe.
    Mon commentaire se voulait simplement logique dans un registre du type "Ladurie, histoire du climat depuis l'an mil", c'est à dire une observation régionale - en est-il autrement au sujet du climat... - démontrant simplement que la "variabilité naturelle" n'a pas disparu!! (la question est posée dans votre billet).

    RépondreSupprimer
    Réponses
    1. Si vous avez vraiment un DESS en génie physique vous êtes effectivement le bienvenu, mais uniquement pour apporter de la valeur ajoutée en vous servant de ce que vous avez appris et pas en remettant simplement en cause 200 ans de science depuis Fourier ; le rôle du CO2 anthropique ne fait plus aucun débat aujourd'hui, tous les autres facteurs sont négligeables, il vous suffit de lire un rapport du GIEC pour le savoir.

      Par conséquent je n'admettrai pas que quelqu'un ici pourrisse le débat en prenant de la place dans les commentaires avec des considérations telles que « si on admet l'hypothèse d'un réchauffement du au CO2 », j'espère m'être bien fait comprendre.

      A la prochaine incartade c'est direct la page poubelle.

      Quant à Ladurie et le climat depuis l'an mil je vous invite à lire mes deux articles sur lui et son livre, c'est ici : http://sogeco31.blogspot.com/2016/04/emmanuel-le-roy-ladurie-et-guillaume.html et http://sogeco31.blogspot.com/2016/05/emmanuel-le-roy-ladurie-et-guillaume.html ; mais aussi : http://sogeco31.blogspot.com/2015/10/il-faut-ecouter-et-lire-emmanuel-le-roy.html ; quant à l'attribution de la hausse de températures aux divers facteurs retenus par le GIEC vous avez le tableau dans cet article : http://sogeco31.blogspot.com/2016/04/gros-problemes-de-comprehension-chez.html ; forçage radiatif anthropique de 1750 à 2011 = 2,29 W m2 (entre 1,13 et 3,33) ; FR dû au soleil : 0,05 W m2 ; comme on dit il n'y a pas photo.

      Enfin je souhaiterais, si vous comptez commenter souvent à l'avenir (de manière positive), que vous apparaissiez sous un pseudo autre que Unknown, ce serait plus sympa pour les autres commentateurs qui voudraient réagir, alors pourquoi ne pas vous enregister sous Hugo, ou Victor ? Vous avez également la possibilité, mais ce n'est pas une obligation, de devenir membre abonné du blog, à vous de voir ce qui vous convient le mieux.

      Supprimer
  8. Un bien curieux message. La science est continuellement "remise en cause", c'est son principe même de fonctionnement et ce qui nous permet d'avancer.
    J'adhère complètement à la démonstration de Geuskens - qui remet complétement "en cause" celle du GIEC - et je ne demande qu'à échanger avec tous ceux qui pourraient "scientifiquement" en débattre.
    Qui ne dit mot consent et essayer de tuer ce débat dans l'œuf par autorité comme vous venez de le faire reviens au même.

    RépondreSupprimer
    Réponses
    1. « La science est continuellement "remise en cause" »

      Où est la remise en cause du rôle du CO2 anthropique dans la littérature scientifique ?

      Vous qui prétendez avoir une formation scientifique vous ne savez donc pas comment fonctionne la démarche scientifique, on a pourtant dû vous apprendre cela à l'école, non ? Ainsi vous donnez de l'importance à un article de blog qui remettrait soi-disant en cause ce que disent la quasi-totalité des climatologues de la planète ? Vous ne connaissez donc pas le processus de revue par les pairs qui, certes, peut parfois avoir des couacs, mais reste le seul outil valable pour se faire une idée de l'état de la science à un moment donné ? Si Geuskens publie un papier dans une revue sérieuse dites-le moi, j'en ferais part dans mon blog, pour l'instant ce n'est que du vent, tout comme le papier toujours en attente de peer review de Susan Crockford sur les ours polaires qui ne semble pas avoir convaincu les relecteurs.

      Enfin ce blog n'est pas un lieu de « débat » étant donné que ni vous ni moi ne sommes compétents, et donc légitimes, pour « débattre » du climat.

      Supprimer
    2. Pour Geuskens, vous avez raté cet article dans lequel VB donnait son avis (c'est ici : http://sogeco31.blogspot.com/2019/02/la-theorie-du-rca-nest-pas-justifiee.html?showComment=1550270349328#c7762063318302736704) : « Georges Geuskens, chimiste spécialisé dans les polymères, a accédé à l'éméritat en 1999, ce qui signifie qu'il devrait avoir aujourd'hui dans les 85 ans. J'ai parcouru son article. C'est clairement écrit par un scientifique qui s'est aventuré trop loin de son domaine d'expertise. Avec de temps en temps une petite pique à l'attention des climatologues qui n'auraient rien compris au sujet. On dirait du Jacques Henry.
      Le problème comme toujours avec ce genre de billet, c'est qu'il peut être cité par des pseudo-sceptiques lambda comme références, avec le bonus que c'est écrit par un Professeur, avec plein de formules compliquées et de x et de y. »

      Nous voyons donc que VB avait touché juste, vous en êtes un parfait exemple.

      Vous pouvez aussi aller plus loin avec ce que pense VB de Geuskens avec http://sogeco31.blogspot.com/2019/02/les-clones-divar-giaever.html?showComment=1550653892515#c5844230594940522254 : « On pourrait vous répondre qu'il appartient aux Geuskens, Giaever, Maurin et autres d'apporter les preuves de ce qu'ils avancent. Ce sont eux qui remettent en question une théorie patiemment élaborée depuis 200 ans, et pour citer Sagan, "extraordinary claims require extraordinary evidences". Un indice, quand quelqu'un qui n'a jamais approfondi un sujet prétend que c'est de la foutaise, ne lui faites pas aveuglément confiance. Et oui, l'âge a une importance - relative. On n'a pas la même acuité intellectuelle, ni surtout la même capacité à assimiler de nouvelles connaissance à 85 ans qu'à 25. Je ne suis pas sûr que vous accepteriez de subir une opération du coeur des mains d'un ORL à la retraite qui a récolté ses informations sur Google et qui traite l'ensemble des cardiologues de charlatans !

      Ceci dit, et puisque vous insistez que nous perdions notre temps à démonter les arguments boiteux de Geuskens, alors que vous pourriez vérifier vous-même la pertinence de ses hypothèses (dans le fond, c'est ce que ferait un sceptique), je vais vous donner des éléments de réponse.

      L'argument de Geuskens est que dans l'atmosphère, aux conditions de pression et de température que l'on connaît, une molécule de CO₂ peut absorber un photon infrarouge, mais délivre tout de suite l'énergie ainsi absorbée par des collisions inélastiques avec les molécules voisines, essentiellement N₂ et O₂. Mais, s'il est vrai que ces échanges d'énergie non-radiatifs ont lieu dans un mélange gazeux à une température donnée, ils s'équilibrent en moyenne. Parce que la molécule de CO₂ ne fait pas que libérer de l'énergie par des collisions, elle en absorbe aussi.
      Par contre, si une molécule de CO₂ peut absorber un photon infrarouge en plus des échanges non-radiatifs, elle peut en libérer un aussi. Geuskens a simplement choisi d'ignorer ce point. Il le dit d'ailleurs lui-même : "Cet équilibre ne sera pas modifié si, en plus, certaines molécules de CO2 sont excitées par absorption d’une fraction du rayonnement infrarouge émis par la Terre". Cette affirmation est tirée de son chapeau, n'est pas argumentée, et il en tire naturellement la conclusion (je paraphrase) : "si j'ignore délibérément que la désactivation du CO₂ peut se faire par rayonnement, j'en conclus que la désactivation du CO₂ ne peut pas se faire par rayonnement". Son explication viole tout simplement le principe de conservation de l'énergie. [à suivre]

      Supprimer
    3. [suite] Quand par la suite il ne comprend pas un raisonnement géométrique relativement simple (à propos de la distribution moyenne d'énergie solaire sur la planète : "Cette valeur n’est pas mesurée mais calculée en admettant une distribution uniforme de l’énergie sur toute la surface terrestre sans tenir compte ni d’un hémisphère non éclairé, ni d’une variation en fonction de la latitude. De ce fait, elle est certainement surestimée"), et qu'en plus il reprend à son sompte l'affirmation que l'augmentation du CO₂ n'est presque pas d'origine anthropique ("Or, de nombreux travaux récents montrent que la teneur en CO2 d’origine anthropique ne serait que d’environ 5 %"), on peut conclure qu'on est en présence d'une personne qui se berce d'illusions. Et je soupçonne que les quelques formules compliquées avec des lettres grecques ne sont là que pour flatter son ego et impressionner le lecteur peu averti.

      Et pour répondre à une de vos questions, si Geuskens proférait ce genre d'inepties, mais "allant dans mon sens", je le critiquerais tout autant, le point important étant le respect de la connaissance et du travail des personnes qui consacrent une vie à étudier un sujet, par rapport à ceux qui les traitent d'ignorants sans savoir de quoi ils parlent. »

      Supprimer
  9. Vous n'avez visiblement pas compris le texte de Geuskens et vous mélangez tout. Vous vous contredisez d'un paragraphe à l'autre et paraphrasez en guise de conclusion;)
    Vous auriez du vous abstenir.

    RépondreSupprimer
    Réponses
    1. « Vous n'avez visiblement pas compris le texte de Geuskens »

      Vous avez des problèmes de lecture ?

      Le texte n'est pas de moi mais de VB, ce serait donc lui « qui mélangerait tout », je pense qu'il va apprécier.

      « Vous auriez du vous abstenir. »

      C'est exactement ce que j'ai envie de vous dire.

      Supprimer
  10. On attend avec impatience les interventions de "VB" et "penseur saumon" :)

    Ah là là.... Fatiguante cette loi de Brandolini ;)

    RépondreSupprimer
    Réponses
    1. Faut pas être impatient, tout vient à point pour qui sait attendre.

      Supprimer
    2. C'était mon intention de rappeler ce que j'avais déjà dit il y a quelques mois sur Geuskens, mais je vois que Ged s'en est chargé avant moi. J'ajouterais que s'il n'y a personne qui a contredit ce brillant Galilée, c'est peut-être parce qu'il a préféré publier ses travaux sur le blog "Science, climat et énergie", que dans Nature ou Science. Il personnifie parfaitement le travers de beaucoup de physiciens âgés (d'autres scientifiques aussi, mais les physiciens sont particulièrement touchés) de prétendre tout savoir dans tous les domaines (https://imgur.com/Rotjqey).

      Il n'y a donc aucun intérêt à discuter du mérite des hypothèses de Geuskens par rapport à la théorie de l'effet de serre. Et la remarque de Daneel est particulièrement pertinente : si quelqu'un produit une hypothèse remettant en cause 200 ans de science bien établie, il doit non seulement apporter de solides arguments en faveur de son hypothèse (quod non, en ce qui concerne Geuskens), mais il doit aussi, et surtout, démontrer en quoi les 200 ans de science bien établie se trompaient.
      Parce qu'il y a, somme toute, un certain paradoxe, dans le chef de Josh, à prétendre que l'effet de serre n'existe pas, mais qu'il est saturé !

      Supprimer
    3. @VB Merci pour ce rappel.
      Puisque Ged vous tient en grande estime, peut etre pourriez vous répondre á ce questionnement que j'ai eu en lisant cet article de Josh.
      Si les feedbacks sont positifs afin d'expliquer l'augmentation de temperature lié au CO2 ( c'est á dire que l'ECS est superieur á l 'effet sec du CO2 - si j'ai bien compris) , comment ce fait il que le climat soit stable avec la variabilité naturelle ? Comment se fait il que la variabilité naturelle ne déstabilise pas le systeme ?
      Et ensuite quel est le mécanisme qui empeche l'emballement ? Parce qu'encore une fois , une boucle globale de retroaction positive devrait etre divergente dans un circuit BF.

      Supprimer
    4. @VB
      Je ne pense pas que Josh nie l'effet de serre, de ce que j'ai compris de ses explications il pense que Dufresne et Bréon n'ont pas la même approche sur le sujet ; que pouvez-vous nous dire sur les pistions de l'un et de l'autre, sont-elles différentes ? Ou bien, ce que je pense personnellement, expliquent-ils la même chose de manière différente ?

      @BenHur
      Votre question n'est pas vraiment le sujet de ce billet qui tente d'adresser le questionnement de Josh, cependant VB peut vous répondre sur ce point ; en ce qui me concerne tout ce que je peux vous dire c'est qu'aux échelles géologiques les paramètres astronomiques (cycles de Milankovic), qui sont par essence naturels, ont provoqué de légères augmentations de températures, lesquelles en provoquant le dégazage en CO2 des océans, ont entrainé des rétroactions fortement positives expliquant les fortes montées de température des sorties de glaciation. Mais le système climatique revient toujours à un certain équilibre, ce que l'on voit notamment depuis quelques milliers d'années pendant lesquelles le climat est resté très stable, ce qui a permis l'émergence des civilisations humaines (à ma connaissance les températures ont varié dans une fourchette d'environ seulement 2°C)

      Je vous dis tout cela de mémoire sans consulter le moindre bouquin, si VB veut me corriger il a ma permission (pas trop fort la fessée svp)

      Supprimer
    5. Lire positions et non pistions...

      Supprimer
  11. Ok .
    Il est tout de meme dommage qu'il n y ait que pour seule réponse aux points de Josh ,á la fois de Bréon et de VB, que "Circulez y a rien á voir !"
    Ces interrogations étaient pourtant exprimées clairement sans animosité , ni esprit de polémique. Comme vous Ged , je ne pense pas que Josh nie l'effet de serre. En plus cela semblait assez pointu ...

    J'en retire tout de meme pour ma connaissance personnelle , la formule issue (et non pas incluse) des modéles donnant une approximation du forcage radiatif en fonction de la concentration de CO2. La réponse de Mr Bréon sur ce point est claire et satisfaisante pour moi (au sens ou je comprend ....). Je continue de penser que ce n'est pas une formule empirique mais c'est du pinaillage.
    Pour le reste, je suis un peu décu ...

    RépondreSupprimer
    Réponses
    1. Pour l'instant le « circulez y a rien à voir » ne concerne que le hors-sujet sur Geuskens amené dans la discussion par Hugo, et j'aurais dû recentrer la discussion sur la question de Josh, c'est ma faute je le reconnais.

      J'attends donc toujours un feedback de VB ou de penseur saumon sur la question précise de Josh qui n'a pour l'instant pas été adressée.

      « Je continue de penser que ce n'est pas une formule empirique »

      Et vous avez tort ; Bréon a bien dit que, je le cite, « la formule donnée par ton contradicteur, Delta F = 6.3 ln (C/Co), est une approximation basée sur des calculs plus complexes qui font intervenir le profil de température de l’atmosphère, la présence de nuage, la vapeur d’eau. J’insiste sur le fait que ce n’est pas qqchose qui est codé dans les modèles de climat, mais qui est déduit de calculs plus complexes », par conséquent cette formule est de toute évidence empirique puisqu'elle n'est pas théorique mais découle d'observations tirées des modèles, elle n'est pas un input de ces modèles et ne sert donc pas à les faire marcher, contrairement aux équations théoriques de la physique et de la chimie.

      Supprimer
    2. Résumons : Josh - et Hugo - tiennent en très haute estime les explications de Geuskens (Josh : "les molécules des gaz à effet de serre sont désactivées par collision avec les molécules de l’air environnant avant d’avoir eu le temps d’émettre un rayonnement infrarouge.", "Le Professeur Geuskens est donc tout à fait dans son domaine de compétence pour traiter ce sujet").

      J'ai déjà expliqué en quoi l'hypothèse de Geuskens est à côté de la plaque. Je veux bien le rappeler brièvement ici : Geuskens ignore tout bonnement le phénomène physique qui lui donne tort. Les molécules dans l'atmosphère, y compris le CO2 échangent en permanence de l'énergie par collisions inélastiques. Echangent, c-à-d en donnent et en reçoivent. Ces échanges sont plus ou moins à l'équilibre. En même temps, et même si les échelles de temps sont différentes, elles peuvent absorber et émettre des photons à l'énergie correspondant à des différences de niveau d'énergie interne. Ces absorptions-émissions sont observées en analysant le spectre du rayonnement infrarouge émis au sommet de l'atmosphère et incident à la surface du sol.

      En exposant ses hypothèses, Geuskens nie explicitement l'effet de serre. Par exemple : "De cet exposé on peut conclure que l’effet de serre, tel que défini en 2a ci-dessus, n’existe pas au niveau des basses couches atmosphériques et que, même s’il existait, l’apport d’énergie résultant de l’absorption d’une fraction du rayonnement thermique de la Terre serait rapidement converti en évaporation de l’eau des océans".

      Donc Josh nie l'effet de serre, Hugo nie l'effet de serre, et toute discussion à ce stade me paraît inutile. Quant aux arguments de Josh, ils me font furieusement à du blabla pseudo scientifique glâné à la fréquentation de sites "sceptiques". J'en prends pour exemple son paragraphe "Remarquons tout d’abord qu’il s’agit d’un problème d’interaction entre un rayonnement électromagnétique et la matière, donc d’un problème de pure physique impliquant le rayonnement du corps noir (Planck), la théorie cinétique des gaz (Boltzmann), la théorie du rayonnement thermique (Kircchoff) et la thermodynamique (Carnot)."
      Quand on comprend que l'absorption ou l'émission d'un photon par un atome ou une molécule n'a rien à voir avec le corps noir, la cinétique des gaz, le rayonnement thermique ou la thermodynamique, on se demande pourquoi Josh a cru bon devoir mentionner tout cela, si ce n'est pour impressionner son auditoire. De plus (paragraphe suivant), bon nombre de climatologues sont physiciens, et comprennent tous ces concepts, mieux que lui ou même que Geuskens. Donc son allusion que "les climatologues ne sont sans doute pas les mieux placés pour en parler" me paraît déplacée. En résumé, Josh montre toutes les caractéristiques du "concern troll" et ne mérite sans doute pas qu'on lui consacre trop de temps.

      Supprimer
    3. @VB, Merci . Si je comprend bien ca n'a pas d'interet de répondre parce que c'est sans interet.

      Comme je l'ai dit je n'ai aucune compétence scientifique , c'est donc difficile pour moi de déterminer si un interlocuteur blablatte ou pas. Mais lá au moins c'est clair.

      Mais il ya quelquechose que je ne comprend pas .
      J'ai cru comprendre que la température d'un gaz était liée á l'agitation de ces molécules. Donc si une partie de l'energie liée á l'absorption des photons dans la bande IR par les molecules CO2, n'est pas transformée en energie cinétique avec les autres molecules , qu'est ce qui réchauffe l'atmosphére ... La je suis perdu .

      Je pensais justement que c'etait cela l'effet de serre : le CO2 absorbe de l'energie et la transmet aux autres molecules sous forme cinétique.

      J'avais également compris que la mesure au sol de la temperature est une mesure de l'atmosphere donc du gaz.

      Supprimer
    4. D'autre part , dans ce que j'avais compris , en donnant cette energie aux autres molecules ( sous forme cinétique) , la molécule de CO2 se trouvait donc disponible de nouveau pour absorber un photon IR .. C'est comme cela que je m'expliquais qu'il n y avait pas d'effet de saturation.

      Supprimer
    5. @BenHur

      Votre question est intéressante, et intelligente, car effectivement savoir comment l'atmosphère est chauffée n'est pas de tout repos tellement j'ai lu d'informations contradictoires sur le sujet.

      Par ailleurs il faut savoir que l'atmosphère est composée de plusieurs couches, je pense donc que votre question porte sur la troposphère uniquement, là où nous vivons, car au-dessus de la tropopause il y a la stratosphère et encore plus haut d'autres couches qui fonctionnent différemment.

      Sur Wikipédia (https://fr.wikipedia.org/wiki/Atmosphère_terrestre) il est écrit que « L'atmosphère protège la vie sur Terre en absorbant le rayonnement solaire ultraviolet, en réchauffant la surface par la rétention de chaleur (effet de serre) et en réduisant les écarts de température entre le jour et la nuit », mais à mon avis cela est très réducteur et ne raconte pas toute l'histoire, notamment celle des gradients de température (verticaux et horizontaux)…

      Je laisse à VB le soin de vous fournir des explications ou de vous orienter vers des lectures sérieuses sur le sujet.

      Supprimer
    6. Arghhh, je viens de me rendre compte que le sujet de la température dans l'atmosphère est justement traité dans le livre que je n'arrête pas de conseiller chaque fois que l'occasion se présente, et là j'ai failli rater l'occasion de vous le recommander ; il s'agit de Climat passé présent futur que vous trouverez à la FNAC ou sur Amazon.

      Pour faire court et en citant des passages du livre page 43, dans la troposphère la convection règne en maitre du fait que l'air est en grande partie transparent au rayonnement solaire ; il n'absorbe que peu ce rayonnement, contrairement à la surface de la planète qui alors se réchauffe. L'atmosphère est ainsi chauffée par le bas, la conduction permet au sol de transmettre la chaleur à la mince couche d'air juste au-dessus, puis la convection prend rapidement le relais etc.

      Je ne vais pas citer tout le chapitre autrement faudrait que je paie des droits d'auteurs;)

      Je vous invite à commander ce livre qui vous expliquera tellement de choses qui ont été à mon niveau et qui ne devraient donc pas vous poser de difficultés.

      Au passage Josh et Hugo feraient bien eux-aussi de se le procurer, cela pourrait leur être utile, je dis ça je dis rien...

      Supprimer
  12. Comme je disais c'est du pinaillage de ma part sur du vocabulaire ;-)
    L'explication de Mr Bréon est claire et ne prete pas á ambiguité (malgré l'emploi du moi empirique, mais qui en fait n'est pas de lui mais une reprise de Josh) et j'ai appris quelquechose d'interessant.

    RépondreSupprimer
  13. Bonjour
    Tout d'abord, je vous remercie pour votre confiance dans mes capacités mais vous m'avez surestimé. En effet, Je suis malheureusement incompétent dans ce domaine de la physique .
    En ce qui concerne l'effet de serre, voila ce que j'ai trouvé qui peut éventuellement apporter de l'eau au moulin de votre discussion :

    Pour ceux qui veulent comprendre ce sujet .

    http://www.climat-en-questions.fr/reponse/fonctionnement-climat/effet-serre-par-philippe-bousquet-jean-louis-dufresne

    L'HDR de JL Dufresne
    http://www.lmd.jussieu.fr/~jldufres/publi/2009/HDR_JLD.pdf
    et
    http://documents.irevues.inist.fr/bitstream/handle/2042/39839/meteo_2011_72_31.pdf

    Un article d'un tout autre style qui vaut quoi ?

    https://arxiv.org/pdf/0707.1161.pdf
    G. Gerlich, R.D. Tscheuchner. Falsification of atmospheric greenhouse effect within the frame of physics, International Journal of Modern Physics-B, vol.23, n° 3 (2009).

    Comme l'on souligné plusieurs internautes, vos articles et votre blog tiennent la route et c'est vraiment intéressant.

    Salutations
    Saumon



    RépondreSupprimer
  14. Merci à VB et penseur saumon d'avoir contribué à « éclairer » Josh, je ne sais pas si celui-ci peut vraiment être qualifié de troll comme le suppose VB (qui a dû en rencontrer pas mal tout comme moi d'ailleurs), j'avais cru comprendre qu'il était de bonne foi et simplement troublé par des lectures...divergentes, mais si c'est bien ce que je pense alors cela aura au moins eu le mérite de nous montrer (ce que nous savions déjà…) que les « marchands de doute » arrivent à polluer de jeunes cerveaux en leur faisant croire qu'il y a un débat parmi les scientifiques.

    J'espère seulement que Josh réfléchira à son tour à la question, non pas à celle qu'il posait, mais à celle qui l'a fait dériver en tenant ce que Bréon qualifiait de propos « confus » qui lui avaient demandé deux lectures avant de décider de ne pas lui répondre.

    Maintenant comme la défense a toujours le dernier mot je laisse à Josh le soin de s'exprimer s'il le désire.

    RépondreSupprimer
  15. Bonjour à tous et merci pour vos commentaires.

    Je cherche à déterminer le mécanisme par lequel le CO2 influe sur le réchauffement climatique, avant, et c’est essentiel, avant prise en compte d’éventuelles rétroactions.
    Dans ses rapports, le GIEC précise le « degré de confiance » pour chaque affirmation, « moyen », « élevé », etc..

    Par exemple, « il est extrêmement probable que les émissions anthropiques de gaz à effet de serre aient été la cause principale du réchauffement observé depuis le milieu du XXe siècle. » ; ce n’est pas certain, mais c’est « extrêmement probable ».

    Je prends la même précaution ; je suis raisonnablement convaincu que ce que je vais expliquer est scientifiquement exact, mais si demain quelqu’un met en doute, preuves à l’appui, certaines de mes conclusions, je l’écouterai avec la plus grande attention.

    M. Geuskens a à la fois raison et tort ; il est manifestement très compétent dans le domaine des interactions entre le rayonnement et la matière ; dans les basses couches de l’atmosphère, la fréquence des collisions entre molécules de l’air ambiant est très élevée ; en conséquence, les molécules de CO2 excitées par le rayonnement infrarouge émis par la terre se désexcitent principalement par collision et non par rayonnement.

    Pour s’en convaincre, il suffit de consulter la figure 2.1 du premier rapport du GIEC (1990) qui montre que, dans la fenêtre d’absorption du CO2 autour de 15 microns, très peu de rayonnement arrive à la tropopause ; ceci confirme bien que la quasi-totalité de ce rayonnement a été absorbée par l’atmosphère.

    Le rayonnement émis par la terre est donc absorbé par quelques dizaines de mètres d’atmosphère, et transformé en énergie cinétique ; tout ajout supplémentaire de CO2, par exemple d’origine humaine, ne créera pas d’absorption supplémentaire, puisque tout est déjà absorbé.

    Contrairement à ce qu’on pourrait croire, le GIEC est à peu près d’accord avec ce constat (voir paragraphe 2 du premier rapport de 1990) , car le mécanisme d’effet de serre qu’il propose est différent ; dans le glossaire du rapport AR5 de 2014, on peut lire :

    « Effet de serre : Effet radiatif de tous les constituants de l’atmosphère qui absorbent le rayonnement infrarouge. Les gaz à effet de serre, les nuages et, dans une moindre mesure, les aérosols absorbent le rayonnement terrestre émis à la surface de la Terre et dans l’atmosphère. Ces constituants émettent un rayonnement infrarouge dans toutes les directions, mais, toutes choses étant égales par ailleurs, la quantité nette de rayonnement émis vers l’espace est alors inférieure que ce qu’elle aurait pu être en l’absence de ces constituants, compte tenu de la baisse de la température avec l’altitude dans la troposphère et de l’affaiblissement de l’émission qui en découle. L’augmentation de la concentration de gaz à effet de serre accroît cet effet; on fait parfois référence à cette différence en utilisant l’expression effet de serre additionnel. L’augmentation de la concentration des gaz à effet de serre découlant d’émissions anthropiques se traduit par un forçage radiatif instantané. La surface terrestre et la troposphère se réchauffent en réponse à ce forçage, rétablissant graduellement l’équilibre radiatif au sommet de l’atmosphère. »

    N’ayez crainte, si vous n’avez rien compris, c’est normal ; la seule explication que j’ai trouvée est celle de M. Dufresne : « l’effet de serre atmosphérique, plus subtil qu’on ne le croit ».

    RépondreSupprimer
  16. M. Dufresne a à la fois raison et tort ; il explique que les molécules de CO2 excitées par le rayonnement infrarouge émis par la terre se désexcitent par rayonnement (et non par collision comme l’affirme M. Geuskens) ; ce rayonnement est absorbé par une autre molécule de CO2 qui va le réémettre à son tour ; il progresse ainsi de couche en couche d’atmosphère jusqu’à arriver à une altitude dite altitude d’émission où, l’air s’étant raréfié, il n’y a plus assez de molécules de CO2 pour capter ce rayonnement, qui est donc émis vers l’espace.

    Lorsque la concentration en CO2 augmente, l’altitude d’émission augmente, et la température diminue ; le rayonnement du CO2 diminue avec la température, donc moins de rayonnement est émis vers l’espace, donc l’atmosphère se réchauffe.

    C’est bien le mécanisme qui est décrit au paragraphe « effet de serre » du 5ème rapport du GIEC ; sauf que, comme on l’a vu précédemment, le rayonnement infrarouge émis par la terre est absorbé avant d’arriver dans les hautes couches de la troposphère où il pourrait être émis vers l’espace.

    La réponse est dans la définition donnée par le GIEC : « Les gaz à effet de serre, les nuages et, dans une moindre mesure, les aérosols absorbent le rayonnement terrestre émis à la surface de la Terre et dans l’atmosphère. »

    Tout corps solide, liquide ou gazeux émet un rayonnement (sauf au zéro degré absolu) ; en l’occurrence, l’atmosphère, même à la tropopause où il fait -50°C, émet un rayonnement : la température est en fait de +223 K, donc suffisamment élevée pour qu’il y ait une agitation thermique significative ; les molécules de CO2 sont donc excitées par collision avec les molécules de l’air environnant ; à basse altitude, comme on l’a vu précédemment, ces molécules se désactivent par collision avec d’autres molécules ; mais l’altitude augmentant, l’air se raréfie, la fréquence des collisions diminue et la désactivation par radiation prend peu à peu le pas sur la radiation par collision ; c’est ce rayonnement, issu de l’atmosphère elle-même, qui (comme prévu par le GIEC) est émis vers l’espace, et non le rayonnement émis par la terre comme expliqué par M. Dufresne.

    Lorsque la concentration en CO2 augmente, l’absorption de ce rayonnement augmente ; l’émission de rayonnement vers l’espace diminue et l’atmosphère se réchauffe.

    Pour ce que j’en sais, M. Geuskens ne mentionne pas ce phénomène, qui est pourtant à l’origine de l’effet de serre décrit par le GIEC ; mais il reste un problème à régler : pour M. Dufresne, comme pour le GIEC, l’altitude d’émission à laquelle l’effet radiatif prend le pas sur les collisions se situe dans la troposphère où la température diminue avec l’altitude ; l’examen de la figure 2.1 du premier rapport du GIEC (déjà mentionnée ci-dessus) montre qu’à la tropopause, le CO2 reste très absorbant ; l’altitude d’émission se situerai donc au-dessus de la tropopause, dans la stratosphère où la température augmente avec l’altitude.

    Lorsque la concentration en CO2 augmente, l’effet de serre en altitude est donc le résultat de deux phénomènes qui se compense partiellement : l’augmentation d’émission avec l’altitude, et la diminution d’émission

    RépondreSupprimer
  17. Reste à mettre des chiffres sur cet impact du CO2 sur l’échauffement de l’atmosphère ; je vais reprendre les chiffres de M. Bréon :

    « Aujourd’hui, on augmente de 2-3 ppm/an. Donc, sur 30 ans, on va se prendre au moins 70 ppm. Donc un forage supplémentaire de 6.3 Ln (480/410)= 1 W.m2»

    Comme déjà mentionné dans un message précédent, en dérivant la loi de Boltzmann, on obtient:

    (Delta T) = T/4 x (Delta F) / F

    où T est la température de la terre, soit 288 °K, Delta F vaut 1 W.m-2, et F est la radiation totale de la terre, soit 238,5 W.m-2; donc l'augmentation de température correspondant au supplément d'absorption par les 70 ppm de CO2 supplémentaires émis dans les trente prochaines années sera:

    288/4 x 1 / 238,5 = 0,3 °C.

    0,3°C en 30 ans cela fait un centième de degré par an ; bien sûr si quelqu’un trouve une erreur dans ce calcul, il peut en proposer un autre, je l’étudierai avec intérêt.

    Je pense que le GIEC a bien compris que le CO2 à lui tout seul ne pouvait pas justifier un réchauffement important ; la raison en est que l’absorption par le CO2 augmente très faiblement avec la concentration ; donc le GIEC met en avant les rétroactions positives ; je me permets de poser une question : avec un centième de degré par an, quelles preuves a-t-on que c’est bien le CO2 qui est à l’origine de ces rétroactions positives ?

    J’ai eu beau chercher, je n’ai pas trouvé la réponse ; si quelqu’un a une idée, il est le bienvenu.

    RépondreSupprimer
  18. Cher Josh ,

    En reprenant vos deux formules et en tentant de retrouver l'ECS correspondant , je trouve un ECS de 1.31 degC . C'est donc en dehors de l'interval de l'ECS fourni par le GIEC [1.5 degC - 4.5 degC] , mais pas si loin.

    Donc votre approche ne semble pas totalement en contradiction avec les données du GIEC si l'on considére que l'ECS est dans la fourchette basse.

    Pour le les 0.3 degC en 30 ans , c'est également pas si loin que ca des observations, du moins on a le bon ordre de grandeur ( trend : 0.15 degC/ dec en observation , mais ca varie lorsqu'on fort El-Nno (en + ou en -) est á une des extrémités de l'interval considéré).

    RépondreSupprimer
    Réponses
    1. Si j'ai bien compris le raisonnement de Josh son calcul est effectué avant toute rétroaction, donc à mon avis il ne s'agit pas de l'ECS (au féminin puisqu'il s'agit de sensibilité) qui, elle, se calcule à l'« équilibre » (le E de ECS) ; la fourchette du GIEC (de 1,5 à 4,5) représente l'ECS, donc si l'on rajoute au chiffre de Josh toutes les rétroactions on entre en plein dedans…

      Par ailleurs El Niño fait partie de la variabilité naturelle du climat, il ne doit donc pas être pris en compte pour l'ECS ou tout autre calcul de sensibilité.

      Supprimer
    2. C'est pour cela que je dis qu'on est pas tres loin . Si on ajoute les retro-actions on doit atteindre aisément la fourchette du GIEC.

      Cela confirme également une information que j'avais lu, je ne sais plus ou : l'effet sec (sans retro-action donc) du CO2 est autour de 1.2 degC . Donc on tombe bien autour de cela. C'est trés rassurant.

      La discussion se situe donc bien autour des rétro-actions et pas forcément sur la quantification de l'effet sec du CO2.

      Je note que je ne vois pas ou est le climato-scepticisme/negationnisme jusqu'ici ....

      Supprimer
  19. Josh,
    Je ne comprends pas votre phrase "le rayonnement du CO2 diminue avec la température", au contraire le nombre de collisions diminuant avec la température, le rayonnement augmente.
    Enfin votre dernier message est en contradiction avec le reste.
    Ces calculs me laissent songeur. Je pense que tout le monde sera d'accord avec: "En intégrant l’équation de Planck de 14 à 16 µm, domaine d’absorption du CO2, on calcule que 9,3 % du rayonnement thermique de la Terre sont émis dans cet intervalle de longueurs d’onde. En fin de compte le CO2, quel que soit son coefficient d’absorption, ne pourrait absorber au maximum que 9,3 % du rayonnement thermique de la Terre (représentant, selon la NASA, 21 % de l’énergie totale reçue du Soleil au sommet de l’atmosphère) soit un peu moins de 2 % de 342 W m-2 donc environ 7 W m2".
    Si l'on reprend le calcul de M. Bréon, une augmentation de 1000 ppm nous donnerait un forçage supplémentaire de 6.3 Ln (1410/410) = 7,78 W.m2
    Donc 1000 ppm (un peu moins en fait) c'est le niveau à partir duquel le CO2 se transformerait en soleil ;)
    Hugo.
    Hugo.

    RépondreSupprimer
    Réponses
    1. Heu non Hugo , ca ne marche pas comme ca ... de mon point de vue.

      Si j'ai bien compris , le C02 modifie les propriétés radiatives de l'atmosphere.
      On a un équilibre radiatif globale de la Terre . Si je comprend bien , la Terre recoit de l'energie durant le jour et en perd durant la nuit ( cycle jour/nuit).
      En modifiant les propriétés radiatives de l'atmosphere , le CO2 ralentit la perte d'energie la nuit . Donc comme les nuits ont toujours la meme durée, le bilan radiatif n'est plus "équilibré" et si on fait la difference on trouve ce "forcage radiatif" qui correspond en fait au rayonnement qui n'a pas été evacué. Cette energie, non évacuée, réchauffe l'atmosphere et finalement cela rééquilibre le bilan radiatif mais avec une autre temperature d'equilibre de l'atmosphere.

      Bon , mon explication est peut etre totalement foireuse mais c'est comme cela que je voiyais les choses

      Supprimer
  20. Le Co2 peut bien ralentir tout ce que vous voudrez, il n'en reste pas moins qu'il ne peut absorber que 7 w.m2 et cela dépend uniquement du rayonnement thermique de la Terre et de l'énergie reçue du soleil.
    D'ailleurs j'en étais resté à la formule ∆F (W m-2) = 5,35 ln C/C0 pour lier le forçage radiative au Co2, mais tout cela reste très empirique et dans un concept de "forçage" uniquement lié au CO2…
    Bref plus raisonnablement personne ne nie un "effet de serre" dans les hautes couches de l'atmosphère ce qui est confirmé par nos satellites. En revanche dans les basses couche la température et la pression sont trop importantes et le CO2 se désactive par collision - la fluorescence du CO2 ne peut pas entrer en compétition. On est d'ailleurs bien incapable d'observer quoi que ce soit comme le cite très justement Josh "la figure 2.1 du premier rapport du GIEC (1990) qui montre que, dans la fenêtre d’absorption du CO2 autour de 15 microns, très peu de rayonnement arrive à la tropopause".
    Hugo.

    RépondreSupprimer
  21. A Hugo, Josh et BenHur,

    Je ne voudrais pas que mon site devienne comme Skyfall, un lieu où des gens qui n'ont aucune compétence sur un sujet se croient quand même autorisés de donner leur avis, surtout si cet avis est en contradiction avec ce que l'on peut lire dans la littérature scientifique sérieuse ; cela vaut surtout pour Josh et Hugo, mais aussi pour BenHur qui, malgré sa bonne volonté évidente, ne possède pas le bagage nécessaire, tout comme moi, pour entrer dans les détails techniques.

    Vous devriez prendre exemple sur penseur saumon qui a avoué ne pas être capable de répondre à la question de Josh et a seulement fourni des éléments pouvant éventuellement permettre de comprendre les processus en jeu, même si la référence à Gerlich et Tscheuchner va embrouiller la réflexion, mais il a bien précisé « un article d'un tout autre style qui vaut quoi ? » (la réponse que VB ferait, à mon avis, est : pas tripette)

    Donc je vous demanderai de ne plus intervenir pour donner votre avis à l'aide de formules que vous ne maitrisez très probablement pas, seul VB est pour moi légitime pour donner davantage de précisions sur ce qui s'est dit ici ou là.

    Si la discussion devait devenir trop « skyfallienne » je me verrais obligé de fermer ce fil de commentaires.

    RépondreSupprimer
  22. Ce commentaire a été supprimé par un administrateur du blog.

    RépondreSupprimer
  23. Ce commentaire a été supprimé par un administrateur du blog.

    RépondreSupprimer
    Réponses
    1. Je pensais pourtant avoir été clair, Hugo vient de dépasser certaines bornes en dérapant sur le verglas de son incompétence, ce qui l'a fait atterrir malencontreusement dans la page poubelle (https://sogeco31.blogspot.com/p/poubelle.html) où ses deux derniers commentaires resteront en vitrine exposés à la vue de tous pendant très très longtemps.

      Supprimer
    2. Ged , je pense que vous avez raison et je crois avoir dit de trés grosses betises dans mon dernier post....
      Il faut avouer que, comme le dit Josh, pour comprendre finement la mécanique de l'effet de serre , il faut avoir de trés bonne compétences dans des domaines assez ariées de la Physique ( thermodynamique, physique de l'atmosphere, physique radiqtive etc etc etc ) . Ce n'est pas de la pédanterie , c'est juste vrai et c'est pourquoi je trouve le commentaire de VB sur ce point assez ....faible.
      Comme je l'ai déja dit , je trouve assez dommage que les gens qui savent , bottent en touche ... Les questions me semblent claires et ouvertes. Il suffit juste d'expliquer. D'autant plus , que comme vous l'avez vous meme fait remarquer , Josh ne tient pas un discours climato-gogo.
      Néanmoins grace á Mr Bréon et á Josh, j'ai compris quelquechose qui me tiraillait depuis longtemps :
      En combinant la formule approximée du forcage radiatif lié au CO2 et la dérivée de loi de Stefan-Bolzmann (passée par un logarithme .. jolie astuce de calcul !!) , on peut retrouver la valeur de l'effet sec du CO2 .( c'est á dire 1.2-1.3 degC) . C'est extremement satisfaisant intellectuellement et c'est infiniment plus convainquant qu'une experience incertaine comme ici (https://www.youtube.com/watch?v=3v-w8Cyfoq8 -- honnetement je ne vois pas comment cela peut marcher ...).
      C'est avec ce type de démonstration que l'on peut clouer le bec aux inepties comme : "Le 2nd Principe de la thermodynamique n'est pas respecté ..."

      Au final , je comprend mieux le lien entre CO2 , forcage radiatif et effet sec. D'apres ce que j'avais lu , meme des modeles tres simples donnent une valeur similaire d'effet sec du CO2. Donc si les modeles incluant les mecanismes détaillés donnent aussi la meme valeur, cela renforce l'idée que ces mécanismes doivent etre justes. Puisque tout le monde est d'accord lá dessus , c'est bien sur les rétro-actions que l'on doit discuter ...

      Et lá , je repose mon probleme : Pour passer de 1.3 degC d'effet sec á un ECS de 4.5 degC, c'est énorme ... C'est un bras de levier 10 fois plus important que l'effet meme du CO2 .. Mais dans ce cas lá , comment le climat etait il stable avant 1850 ? La moindre perturbation aurait du le faire diverger ? C'est ca que j'aimerais comprendre ....

      PS : Quel est le background de VB pour que vous le teniez en si haute estime ?

      Supprimer
    3. Autant je suis prêt à passer du temps à expliquer des concepts compliqués à des gens qui ont envie de les comprendre, autant je réchigne à perdre du temps à expliquer des concepts simples à des gens qui n'ont pas envie de les comprendre.

      Josh a coché toutes les cases du pseudo-sceptique bien informé, celui qui a passé du temps sur tous les forums sceptiques et en connaît tous les talking points.

      Eût-il consacré la moitié de son énergie à lire un ouvrage de base de physique de l'atmosphère, il aurait eu les réponses à toutes ses questions. Malheureusement, ces réponses ne lui conviennent pas.

      Quelques exemples ?

      - "les molécules de CO2 n’ont pas le temps de se désactiver par rayonnement, elles se désactivent par collision avec les molécules environnantes ; tout ça tient la route, ce mécanisme me paraît le plus vraisemblable"

      C'est ce que prétend Geuskens, c'est ce que prétend Josh, c'est ce que prétend Hugo, et c'est faux. L'effet de serre se produit à toutes les altitudes et toutes les densités. La différence étant qu'à haute altitude, quand la densité rend moins probable l'absorption d'un photon IR par une molécule de CO₂, l'énergie finit par s'échapper dans l'espace. Voir par exemple Raymond Pierrehumbert

      - "Il reste un fait qui est admis par les deux parties, sceptiques ou non : l’absorption du rayonnement infrarouge par le CO2 est saturé ; ceci a été mis en évidence par Angström, et n’est pas contesté"

      Non, voir encore Raymond Pierrehumbert : Saturation Fallacies

      - "la dernière que j’ai lu explique que c’est corrélé à la vitesse du soleil par rapport au centre de gravité du système solaire"

      Le dada de Scafetta et al. ne tient pas la route, voir par exemple sketical science. A tout bien considérer, penser que la précession de l'orbite de Jupiter influence maintenant notre climat, alors que jusqu'à présent ce n'était pas le cas, me paraît assez désespéré.

      - "imaginer que la tropopause à -19°C puisse réchauffer la basse atmosphère à 14°C est contraire au deuxième principe de la thermodynamique : un corps froid ne peut pas céder de chaleur à un corps plus chaud que lui."

      Alors là c'est le pompon, on est dans la catégorie Jacques Henry/Jean-Pierre Bardinet. Je ne m'étendrai pas là dessus.

      Supprimer
    4. "t lá , je repose mon probleme : Pour passer de 1.3 degC d'effet sec á un ECS de 4.5 degC, c'est énorme ... C'est un bras de levier 10 fois plus important que l'effet meme du CO2 .. Mais dans ce cas lá , comment le climat etait il stable avant 1850 ? La moindre perturbation aurait du le faire diverger ? C'est ca que j'aimerais comprendre ...."

      L'effet du CO2 seul ne suffit pas à perturber le climat. Mais le CO2, contrairement à la vapeur d'eau, est un gaz qui ne condense pas. Il reste dans l'atmosphère. C'est pourquoi sa concentration a une telle importance.

      Je vous suggère de lire le papier de Lacis et al.

      Même s'il ne répond pas à toutes vos interrogations, c'est un point de départ intéressant pour une recherche plus approfondie.

      Supprimer
    5. Quant à mon background, je pense que Ged connaît mon nom et mes qualifications.

      J'ai une formation scientifique avancée en physique et j'ai fréquenté professionnellement des climatologues depuis plus de 20 ans.

      Ceci dit, n'étant pas moi-même climatologue, je me repose sur les travaux de gens qui en savent plus que moi. Je me sens par contre parfois habilité à critiquer les écrits de gens qui manifestement vont au delà de leur sphère de compétence.

      Supprimer
    6. Merci pour vos commentaires; j'ai mis un peu de temps à lire les articles que vous citez, que j'ai trouvé fort intéressants; par exemple dans l'article de M. Pierrehumbert, je relève:

      "Coupled vibrational and rotational states are the key players in IR absorption. An IR photon absorbed by a molecule knocks the molecule into a higher-energy quantum state.
      Those states have very long lifetimes, characterized by the spectroscopically measurable Einstein A coefficient. For example, for the CO2 transitions that are most significant in the thermal IR, the lifetimes tend to range from a few milli -
      seconds to a few tenths of a second. In contrast, the typical time between collisions for, say, a nitrogen-dominated atmosphere at a pressure of 104 Pa and temperature of 250 K is well under 10−7 s. Therefore, the energy of the photon will almost always be assimilated by collisions into the general energy pool of the matter and establish a new Maxwell–Boltzmann distribution at a slightly higher temperature. That is how
      radiation heats matter in the LTE limit."

      Bon, jusque là, il est tout à fait d'accord avec M. Geuskens; mais il ajoute:

      According to the equipartition principle, molecular collisions maintain an equilibrium distribution of molecules in higher vibrational and rotational states. Many molecules occupy those higher-energy states, so even though the lifetime of the excited states is long, over a moderately small stretch of time a large number of molecules will decay by emitting photons. If that radiation escapes without being reabsorbed,
      the higher-energy states are depopulated and the system is thrown out of thermodynamic equilibrium. Molecular collisions repopulate the states and establish a new thermo -
      dynamic equilibrium at a slightly cooler temperature. That is how thermal emission of radiation cools matter in the LTE limit.

      La, il n'est plus du tout d'accord avec Geuskens, et j'avoue ne pas comprendre sa logique: si les collisions sont entre dix mille et dix millions de fois plus fréquentes que les radiations, bien sûr, il y a quelques désexcitations par rayonnement, entre une sur dix mille et une sur dix millions; mais le photon qui est émis va être absorbé par la molécule suivante, qui a la même probabilité de désexcitations, soit entre dix mille au carré et dix millions au carré.

      Le libre parcours moyen de ces photons me paraît assez court, et c'est ce que pense Geuskens; pouvez-vous m'expliquer pourquoi vous êtes d'accord avec Pierrehumbert et pas avec Geuskens ?

      Que veut dire: "If that radiation escapes without being reabsorbed" ? c'est possible bien sûr dans la haute atmosphère, mais pas dans les conditions initialement décrites:

      "a nitrogen-dominated atmosphere at a pressure of 104 Pa and temperature of 250 K".

      Je lirai vos commentaires avec beaucoup d'intérêt.

      Supprimer
  24. Bon je pense que maintenant c'est clair pour tout le monde, mais si cela ne l'est pas pour certains je les invite à retourner sur Skyfall où leurs connaissances scientifiques seront mises à profit.

    Je remercie VB pour ses explications et le temps qu'il a passé pour tenter d'éduquer deux de mes commentateurs, probablement en pure perte, mais il aura tout de même essayé.

    Merci également à penseur saumon pour avoir fait ce qu'il a pu en étant honnête sur son manque de connaissances, ainsi qu'à BenHur dont les questions n'étaient pas si idiotes que ça (je lui suggère cependant de se documenter avec l'ouvrage que je lui ai conseillé, cela devrait suffire pour avoir une bonne idée du sujet)

    On peut donc fermer ce fil de discussions et passer à autre chose.

    RépondreSupprimer
  25. VB : "J'ai une formation scientifique avancée en physique "..."L'effet de serre se produit à toutes les altitudes et toutes les densités".
    GED : "On peut donc fermer ce fil de discussions et passer à autre chose"
    Voilà ce qui s'appelle faire autorité sans aucune un argumentation scientifique.
    Amen.

    RépondreSupprimer
  26. Cher VB , je sens que je vous ai froissé et je vous présente sincerement mes excuses.
    J'ai comme postulat que la théorie et ses mécanismes sont clairement définis et cohérents. J'essaye juste de les comprendre. ET c'est en les comprenant que je peux déconstruire ( je prefere á debunker) les arguments climato-gogos.
    Je prend l'exemple de la non-saturation de l'effet de serre. Ce qui m'interesse c'est de comprendre pourquoi il n y a pas saturation . En aucun cas je ne le remet en cause. ( Ce n'est pas parce que je ne comprend pas la mecaQ que je remet en cause la theorie de la mécaQ) . Et d'autre part cela me permet de déconstruire un des arguments climato-sceptique phare qui est que l'effet de serre est saturée ( lá je comprend bien leur objectifs et pourquoi ils insistent dessus).

    D'ou ma frustration sur ce post. Peu importe que Josh soit un troll informé ou non (pour ma part je ne le pense pas ) , ce qui m'aurait interessé c'etait la déconstruction de son raisonnement.

    Cordialement

    RépondreSupprimer
    Réponses
    1. « Peu importe que Josh soit un troll informé ou non (pour ma part je ne le pense pas ) »

      Moi non plus je ne pense pas que Josh soit ce qu'on appelle un troll, il est simplement un malade atteint du syndrome de Dunning-Kruger, quand il aura pris conscience de sa maladie il pourra enfin se soigner.

      Supprimer
  27. Sinon je suis bien d'accord avec vous sur les points suivants que vous avez relevé :
    - 2ene principe de thermo non respecté --> risible
    - Precession de Jupiter influencant le climat ( le "fameux" cycle de 60 ans) --> pas crédible ( il y a aussi un festival de chant letton uniquement tous les 60 ans ... peut etre que cela joue aussi ...)

    Pour la non-saturation , je vais me documenter pour comprendre et lire le lien que vous avez donné. C'est important pour de comprendre pourquoi et ou l'argument climatosceptique est faux

    Pour Geuskens, je suis un peu perdu parce que je ne vois pas en quoi cela remet en cause l éffet de serre .... Si je comprend bien son raisonnement , la couche basse se rechauffe par absorption du rayonnement (et transformation en energie cinétique) puis avec la convection j'imagine que cette chaleur est portée en altitude ou le rayonnement permet d'evacuer cette chaleur. La aussi je vais lire et tacher de comprendre quelle sont les conséquences de la "theorie" de Geuskens ...

    RépondreSupprimer
    Réponses
    1. J'ai trouvé ce lien qui peut vous intéresser, il date de 2015 et est donc très récent : http://www.climat-en-questions.fr/reponse/fonctionnement-climat/effet-serre-par-philippe-bousquet-jean-louis-dufresne

      C'est assez technique, un peu trop pour moi et certainement pour Hugo, même si Dufresne et Bousquet écrivent vers la fin que « Le calcul du bilan radiatif de la Terre en réponse à une modification de la concentration en dioxyde de carbone repose sur des bases physiques solides et ne comporte plus de difficulté fondamentale » ; le principe est donc acquis pour les physiciens qui ne discutent plus les « fondamentaux » qui nous disent que « on voit que l’effet de serre augmente bien, si la concentration en gaz à effet de serre augmente, le tout ayant pour conséquence un réchauffement de la basse atmosphère ».

      La « basse atmosphère » pour moi c'est la troposphère dans sa totalité, car il me semble avoir compris que le reste (stratosphère et au-delà) n'était pas pris en compte, d'ailleurs les auteurs disent bien « Le modèle présenté ici comporte un certain nombre de simplifications : pas de prise en compte de la stratosphère où le gradient thermique est positif à cause de l’absorption du rayonnement solaire par l’oxygène et surtout par l’ozone (on montre que la stratosphère se refroidit lorsque le CO2 augmente) »

      Supprimer
    2. Je confirme que quand on parle de « basse atmosphère » il s'agit bien uniquement de la troposphère, voir : http://files.meteofrance.com/files/glossaire/FR/glossaire/designation/98_curieux_view.html#100

      Supprimer
  28. L'effet de serre mon cher benhur c'est le rayonnement du CO2. Geuskens explique très bien que ce rayonnement n'est pas permis dans les basses couches atmosphériques et les physiciens avancés de ce blog (et ailleurs) non visiblement aucun d'argument à faire valoir pour prouver le contraire.
    Même si tout cela est bien contraire aux théories du GIEC ça ne veut pas non plus dire qu'il n'y a pas de RCA.

    RépondreSupprimer
    Réponses
    1. « L'effet de serre mon cher benhur c'est le rayonnement du CO2 »

      Arrêtez de vous ridiculiser et d'encombrer inutilement ce fil de discussions ou bien vous aller retrouver vite fait la poubelle.

      L'effet de serre c'est bien plus que le rayonnement du CO2, si vous en êtes toujours là c'est à désespérer.

      « théories du GIEC »

      Combien de fois faudra-t-il répéter que le GIEC n'est pas un chercheur et n'a pas de théorie !

      Vous êtes de toute évidence totalement inculte en la matière et même moi simple comptable j'ai l'impression d'en savoir dix fois plus que vous, pourtant j'ai énormément de lacunes.

      Supprimer
  29. Ce qui ne fonctionne pas sur ce blog c'est de considérer qu'il n'y a aucune coquille dans les théories du GIEC ou dans les publications de Nature. Les faits nous prouvent le contraire.
    Les concepts du type forçage radiatif du CO2 avec une formule qui ne repose que sur une constante et une variable sont évidemment trop simpliste et risible. Comme montré plus haut ça fonctionne très bien avec 100 ppm (ce pourquoi il a été conçu), mais au delà on arrive très vite à des aberrations.

    RépondreSupprimer
    Réponses
    1. « considérer qu'il n'y a aucune coquille dans les théories du GIEC »

      A ma connaissance la seule coquille détectée dans un rapport du GIEC concerne la fonte totale prévue des glaciers de l'Himalaya (2350 au lieu de 2035 comme il avait été écrit), vous en connaissez d'autres ?

      Pour vous paraphraser moi je dirais « Ce qui ne fonctionne pas sur Skyfall/WUWT/Mythes et Mancies/etc. c'est de considérer qu'il n'y a aucune coquille dans les théories farfelues de Geuskens/Svensmark/Lindzen/Curry/etc. ou dans les publications du Heartland Institute. Les faits nous prouvent le contraire. »

      Supprimer
  30. Vous avez raison GED, l'effet de serre c'est quand on plante des tomates sous une bâche.

    RépondreSupprimer
    Réponses
    1. Vous venez enfin de dévoiler aux yeux de tous pourquoi vous ne pigez rien du tout à l'effet de serre, merci beaucoup !

      Supprimer
  31. "moi simple comptable j'ai l'impression d'en savoir dix fois plus que vous, pourtant j'ai énormément de lacunes"
    C'est exactement ça, ce n'est qu'une impression et vous avez énormément de lacunes.

    RépondreSupprimer
    Réponses
    1. C'est beau quand on a l'impression d'être un surhomme qui a tout compris, surtout que l'immense majorité des scientifiques de la planète sont des crétins qui ne comprennent rien et ne racontent que des bêtises.

      Vous avez l'air d'être un crack en matière de logarithmes népériens, un peu d'humilité par-dessus ne vous ferait pas de mal, car se considérer comme un dieu nuit gravement à la santé mentale.

      Supprimer
  32. Concernant Geuskens, les faits sur ce blog nous montre surtout votre incapacité à prouver (un blabla autoritaire n'est rien d'autre que botter en touche) une quelconque coquille.

    RépondreSupprimer
  33. Contrairement à vous je ne suis ni croyant ni en mission prosélyte et je cherche justement ou pourrait se trouver la coquille chez Geuskens. Malheureusement ce n'est pas chez vous que je la trouverai.

    RépondreSupprimer
    Réponses
    1. Allez sur Skyfall et demandez-leur, ils vous diront (qu'il n'y a aucune coquille) preuves à l'appui !

      Maintenant si vous trouvez un site sérieux (pas comme le mien hein !) qui trouve et explique la coquille de Geuskens merci de nous en informer.

      Supprimer
    2. Au fait, VB vous a parfaitement répondu, voir plus haut son commentaire dans lequel il écrit « Geuskens ignore tout bonnement le phénomène physique qui lui donne tort. Les molécules dans l'atmosphère, y compris le CO2 échangent en permanence de l'énergie par collisions inélastiques. Echangent, c-à-d en donnent et en reçoivent. Ces échanges sont plus ou moins à l'équilibre. En même temps, et même si les échelles de temps sont différentes, elles peuvent absorber et émettre des photons à l'énergie correspondant à des différences de niveau d'énergie interne. Ces absorptions-émissions sont observées en analysant le spectre du rayonnement infrarouge émis au sommet de l'atmosphère et incident à la surface du sol.

      En exposant ses hypothèses, Geuskens nie explicitement l'effet de serre. Par exemple : "De cet exposé on peut conclure que l’effet de serre, tel que défini en 2a ci-dessus, n’existe pas au niveau des basses couches atmosphériques et que, même s’il existait, l’apport d’énergie résultant de l’absorption d’une fraction du rayonnement thermique de la Terre serait rapidement converti en évaporation de l’eau des océans". »

      Je ne pense pas que VB ait l'intention de perdre son temps avec vous en développant davantage.

      Supprimer
  34. Ged, une formule avec une multiplication et un ln c'est niveau BAC...

    RépondreSupprimer
    Réponses
    1. Voilà, encore une fois vous venez de vous dévoiler, vos raisonnement sont de niveau bac, merci de nous avoir éclairés.

      Supprimer
  35. C'est votre blog, vous êtes chez vous et vous avez parfaitement le droit de décider qui peut intervenir et qui mettre à la poubelle.

    Pour ma part, j'ai toujours pu m'exprimer librement et je vous en suis reconnaissant.

    Je vous rappelle que mon but est de déterminer le mécanisme par lequel le CO2 influe sur le réchauffement climatique, avant, et c’est essentiel, avant prise en compte d’éventuelles rétroactions.

    Le glossaire du 5ème rapport du GIEC précise que l'effet de serre est du à la réduction du "rayonnement émis vers l’espace ...compte tenu de la baisse de la température avec l’altitude dans la troposphère et de l’affaiblissement de l’émission qui en découle."

    Sauf que, comme on l'a vu, la figure 2.1 du premier rapport du GIEC montre qu'à la tropopause, le CO2 est encore à peu près 100 % absorbant; à cette altitude, il n'y a donc pas, ou très peu de "rayonnement émis vers l'espace", puisqu'il est encore absorbé par le CO2 (dans sa bande de fréquence d'absorption).

    Comme je l'ai expliqué dans un message précédent, l'altitude d'émission, à laquelle la désactivation des molécules de CO2 se fait plus par radiation que par collision, se situerait donc dans la stratosphère, et non dans la troposphère.

    Comme déjà expliqué, dans la troposphère, la température augmente avec l'altitude; une augmentation de la concentration en CO2 qui provoque une augmentation de l'altitude d'émission provoquerait donc une augmentation de la température, donc une augmentation de l'émission du CO2 vers l'espace, c'est à dire un refroidissement de la planète (le contraire de ce qui se passerait si l'altitude d'émission se trouvait dans la troposphère).

    Dans ces conditions, la formule Delta F = 6,3 ln (C/Co) n'est plus valable; il s'agit d'un calcul beaucoup plus compliqué de la variation de la fréquence d'émission du CO2 en fonction de la température, que je suis incapable de faire; mais le résultat est clairement que la planète se refroidirait lorsque la concentration en CO2 augmente.

    J'ai déjà écrit à de nombreuses reprises qu'il était illusoire de vouloir prévoir quelle serait la température de la planète en 2050 ou en 2100 sans avoir au préalable déterminé par quel mécanisme le CO2 influe sur le réchauffement; en voilà un bon exemple.

    Les éléments que je cite ne viennent pas d'un article des sceptiques, ils ont été tous deux publiés dans un rapport du GIEC; ce problème doit donc être résolu avant de commencer à écrire les modèles mathématiques décrivant l'évolution du climat; je l'ai déjà soumis sans succès à M. Bréon, mais je suis disposé à en discuter à nouveau.

    RépondreSupprimer
    Réponses
    1. « Comme déjà expliqué, dans la troposphère, la température augmente avec l'altitude »

      J'imagine qu'il s'agit d'une coquille de votre part…

      « J'ai déjà écrit à de nombreuses reprises qu'il était illusoire de vouloir prévoir quelle serait la température de la planète en 2050 ou en 2100 »

      C'est là où vous montrez vos limites, personne n'a jamais prétendu « prévoir » quelle serait la température en 2050 ou 2100, il est urgentissime que vous appreniez les bases de la climatologie, le livre Climats passé présent futur peut vous y aider.

      Supprimer
    2. Oui, vous avez raison, je voulais dire stratosphère et j'ai écrit troposphère; bravo pour votre sagacité.

      Suite à vos conseils, j'ai finalement trouvé un exemplaire du livre Climats passé présent futur; il ne traite pas du tout du problème qui m'intéresse; il n'est par exemple pas fait mention du mécanisme décrit par le GIEC et M. Dufresne concernant l'altitude d'émission; je ne veux pas apprendre la climatologie, je le répète encore une fois, je veux seulement savoir par quel mécanisme le CO2 influe sur le climat; le livre que vous conseillez dit en substance :"par absorption du rayonnement tellurique"; merci, mais ça ne m'aide pas beaucoup.

      Le travail du GIEC est de prévoir quelle sera la température en 2050 et en 2100: paragraphe 2.2 du 5ème rapport:

      "Vers la fin du XXIe siècle (2081–2100), le réchauffement moyen à la surface du globe par rapport à la période 1850–1900 sera probablement supérieur à 1,5 °C selon les scénarios RCP4,5, RCP6,0 et RCP8,5 (degré de confiance élevé), et probablement
      supérieur à 2 °C selon les scénarios RCP6,0 et RCP8,5 (degré de confiance élevé). Il est plus probable qu’improbable qu’il soit supérieur à 2 °C selon le scénario RCP4,5 (degré de confiance moyen). Il est improbable qu’il soit supérieur à 2 °C selon le
      scénario RCP2,6 (degré de confiance moyen). {2.2.1}

      Vers la fin du XXIe siècle (2081–2100), le réchauffement moyen à la surface du globe par rapport à la période 1986–2005 aura atteint probablement entre 0,3 °C et 1,7 °C selon le RCP2,6, entre 1,1 °C et 2,6 °C selon le RCP4,5, entre 1,4 °C et 3,1 °C
      selon le RCP6,0 et entre 2,6 °C et 4,8 °C selon le RCP8,59. L’Arctique continuera de se réchauffer plus rapidement que l’ensemble du globe (figure RID.6a, figure RID.7a). {2.2.1, figure 2.1, figure 2.2, tableau 2.1}"

      Je ne peux que répéter qu'il n'est pas possible de prévoir la température en 2100 sans savoir par quel mécanisme le CO2 agit sur le climat; vous pensez que je suis malade, c'est votre droit.

      Par exemple, si, comme le montre la figure 2.1 du premier rapport du GIEC, l'altitude d'émission se trouve dans la stratosphère, alors la température de la planète baisse quand la teneur en CO2 augmente (voir explication dans mon précédent message); vous n'êtes pas d'accord ? vous avez le droit, j'attends vos contre arguments.

      Supprimer
    3. « vous pensez que je suis malade »

      Je n'ai jamais dit que vous étiez malade, je pense avoir simplement ironisé sur le syndrome Dunning-Kruger qui serait une maladie dont vous seriez atteint, ce qui me semble assez évident à vous lire, mais le syndrome DK n'étant pas une véritable maladie vous êtes par conséquent parfaitement sain de corps (du moins j'espère) et d'esprit !

      Je suis étonné que vous ne trouviez pas dans le livre Climats etc. ce que vous recherchez, le dioxyde de carbone occupe une place non négligeable dans l'index, êtes-vous sûr d'avoir cherché partout ? Cependant ce livre est généraliste malgré quelques passages plutôt ardus pour les profanes, il est donc possible qu'il n'entre pas suffisamment dans les détails qui vous intéressent, mais en fait je n'ai toujours pas compris ce que vous recherchez vraiment, si c'est de remettre en cause la science de l'atmosphère des deux derniers siècles pourquoi pas, mais dans ce cas il vous faudrait lire autre chose que du Geuskens pour avancer autrement vous allez vous embrouiller plus qu'autre chose ; peut-être que quelques cours spécifiques de physique/chimie prodigués par de véritables spécialistes pourraient vous aider ?

      En ce qui me concerne je n'ai pas de contre-argument à vous proposer, il me semble que c'est plutôt à vous d'avancer vos propres contre-arguments si vous voulez réfuter Dufresne ou quiconque d'autre dont le GIEC s'inspirerait, pour cela vous savez ce qui vous reste à faire.

      Supprimer
  36. Josh vous dites :
    "Comme je l'ai expliqué dans un message précédent, l'altitude d'émission, à laquelle la désactivation des molécules de CO2 se fait plus par radiation que par collision, se situerait donc dans la stratosphère, et non dans la troposphère."

    Pourquoi les collisons ne feraient que désactiver les molecules de CO2 ? Elles peuvent aussi les activer en leur donnant de l'energie, non ? (C'est ma compréhension de l'equilibre rayonnement/collision dont parle VB). Et ensuite elles peuvent rayonner.

    RépondreSupprimer
  37. Oui, exactement; les molécules de CO2 sont activées par collision avec les molécules de l'air environnant; à basse altitude, elles se désactivent par collision, mais lorsque l'air se raréfie, la fréquence des collisions diminue, et elles peuvent se désactiver par rayonnement; lorsque ce rayonnement est émis vers l'espace, la terre se refroidit; lorsque la concentration en CO2 augmente, l'altitude d'émission augmente, la température baisse (si on est dans la troposphère), donc le rayonnement diminue et la terre se refroidit moins: c'est l'effet de serre tel que décrit dans le glossaire du GIEC.

    RépondreSupprimer
  38. @Ged "Voilà, encore une fois vous venez de vous dévoiler, vos raisonnement sont de niveau bac, merci de nous avoir éclairés" j'ai repris la formule de VB, il appréciera;) C'est bien vous commencez à découvrir certaines réalités.

    @Josh "lorsque la concentration en CO2 augmente, l'altitude d'émission augmente...". En principe pas puisque l'altitude d'émission ne dépend que de la pression et de la température et non de la concentration en CO2.

    RépondreSupprimer
    Réponses
    1. « j'ai repris la formule de VB, il appréciera;) »

      Je sais bien que vous avez repris la formule de VB, si elle est de niveau bac pas étonnant que vous la compreniez à peu près bien.

      Supprimer
  39. Pour Unknown,

    Oui, je suis un peu d'accord avec vous, mais je ne fais que citer le glossaire du GIEC et l'article de Dufresne; pour eux, l'augmentation de l'altitude d'émission avec l'augmentation de la concentration en CO2 est la base de l'effet de serre.

    RépondreSupprimer
    Réponses
    1. « l'augmentation de l'altitude d'émission avec l'augmentation de la concentration en CO2 est la base de l'effet de serre. »

      Cette phrase n'a pour moi aucun sens…

      La « base » de l'effet de serre est à mon humble avis que quand il y a des gaz à effet de serre dans une atmosphère celle-ci est alors plus chaude que ce qu'elle serait sans ces gaz.

      Supprimer
    2. Pour le coup nous sommes doublement d'accord.
      Hugo.

      Supprimer
  40. Pour ceux qui sont vraiment intéressés voici un lien extrêmement utile : https://history.aip.org/climate/index.htm#contents

    Il s'agit du site d'un auteur (Spencer Weart : https://history.aip.org/climate/author.htm) dont j'ai lu le livre il y a quelque temps ; ce site est bien plus détaillé que le livre, donc pour ceux qui trouveraient les détails trop compliqués le livre fera l'affaire (je ne pense pas qu'il ait été traduit en français)

    Le lien intéressant en ce qui concerne le CO2, l'effet de serre et la soi-disant saturation est ici : https://history.aip.org/climate/co2.htm ; on y parle notamment d'Angstrom, extrait : « The greenhouse effect will in fact operate even if the absorption of radiation were totally saturated in the lower atmosphere. »

    Evidemment il faut lire tout le reste qui explique ce passage.

    RépondreSupprimer
  41. Oui, c'est vrai, c'est ce que j'essaye d'expliquer dans tous mes messages: le rayonnement infrarouge émis par la terre est entièrement absorbé par le CO2 dans les cent premiers métres; ce n'est pas lui qui crée l'effet de serre, c'est le rayonnement crée dans l'atmosphère par excitation des molécules de CO2 avec les molécules d'air environnant; une part de ce rayonnement est émis vers l'espace; c'est une perte d'énergie pour l'atmosphère, qui se refroidit; une augmentation de la teneur en CO2 absorbe un peu de ce rayonnement, donc réduit la fuite de rayonnement vers l'espace , ce qui réchauffe l'atmosphère; c'est ça l'effet de serre décrit par le GIEC; M. Geuskens n'a pris en compte que le rayonnement émis par la terre; M. Dufresne aussi d'ailleurs; ils ont tort tous les deux, et le GIEC a raison.

    RépondreSupprimer
  42. Pour Hugo et Ged

    « l'augmentation de l'altitude d'émission avec l'augmentation de la concentration en CO2 est la base de l'effet de serre. »

    Extrait du glossaire du 5ème rapport du GIEC:

    "Effet de serre: Effet radiatif de tous les constituants de l’atmosphère qui absorbent le rayonnement infrarouge. Les gaz à effet de serre, les nuages et, dans une moindre mesure, les aérosols absorbent le rayonnement terrestre émis à la surface de la Terre et dans l’atmosphère. Ces constituants émettent un rayonnement infrarouge dans toutes les directions, mais, toutes choses étant égales par ailleurs, la quantité nette de rayonnement émis vers l’espace est alors inférieure que ce qu’elle aurait pu être en l’absence de ces constituants,
    compte tenu de la baisse de la température avec l’altitude dans la troposphère et de l’affaiblissement de l’émission qui en découle. L’augmentation de la concentration de gaz à effet de serre accroît cet effet; on fait parfois référence à cette différence en utilisant l’expression effet de serre additionnel. L’augmentation de la concentration des gaz à effet de serre découlant d’émissions anthropiques se traduit par un forçage radiatif instantané. La
    surface terrestre et la troposphère se réchauffent en réponse à ce forçage, rétablissant graduellement l’équilibre radiatif au sommet de l’atmosphère."

    Voila pourquoi j'ai écrit que « l'augmentation de l'altitude d'émission avec l'augmentation de la concentration en CO2 est la base de l'effet de serre":

    "compte tenu de la baisse de la température avec l’altitude dans la troposphère et de l’affaiblissement de l’émission qui en découle. L’augmentation de la concentration de gaz à effet de serre accroît cet effet"

    C'est mieux expliqué dans l'article de M. Dufresne:

    "Que se passe-t-il lorsque la quantité de gaz absorbant augmente ? Du fait de l’augmentation de l’absorptivité, un rayonnement émis à une altitude donnée est absorbé sur des distances plus courtes que précédemment, comme schématisé figure 9b. Le rayonnement émis vers le haut au milieu de l’atmosphère, qui était précédemment partiellement absorbé avant d’atteindre le sommet de l’atmosphère, l’est maintenant totalement. La partie de l’atmosphère qui voit l’espace se réduit par le bas tandis que la partie aveugle de l’atmosphère augmente. En conséquence, l’altitude d’émission augmente. C’est le point crucial."

    C'est M. Dufresne qui dit que c'est le point crucial, moi j'ai seulement dit que c'est la base, même si je ne suis pas trop convaincu par la démonstration.

    RépondreSupprimer
    Réponses
    1. La « base » c'est ceci : « Effet de serre: Effet radiatif de tous les constituants de l’atmosphère qui absorbent le rayonnement infrarouge. »

      Le reste ce sont des détails, certes importants, mais des détails ; sans la base les détails ne sont rien, ils n'existent même pas.

      Supprimer
  43. Je suppose que vous avez quand même compris ce que je voulais dire: pour le GIEC et pour M. Dufresne, l'altitude d'émission augmente avec la concentration en CO2; à plus haute altitude, la température est plus basse donc l'émission du CO2 est plus faible; de ce fait, il y a moins de rayonnement émis vers l'espace , donc l'atmosphère se réchauffe; pour M. Dufresne, "c'est le point crucial" du mécanisme de l'effet de serre (valable aussi pour les autres gaz à effet de serre).

    RépondreSupprimer
  44. Et quelle est le role de la convection ?

    RépondreSupprimer
  45. Bonjour, Je me permets d'intervenir dans cette très longue discussion. Je n'ai pas encore eu le temps de tout lire, vous me pardonnerez. Si j'essaie de résumer la discussion, il est question d'effet de serre radiatif, de transfert radiatif de l'atmosphère et de ce qu'il se passe au niveau moléculaire. Ces questions m'intéressent aussi, et j'ai longtemps cherché une source qui pourrait m'éclairer, et j'ai trouvé ceci : https://www.hindawi.com/journals/ijas/2013/503727/ L'article est de Hermann Harde, qui est bien connu chez des climato-sceptiques puisqu'il a publié des papiers où il prétend démontrer que l'augmentation du CO2 dans l'atmosphère n'est pas anthropique. Mais cet article-ci étudie le transfert radiatif de l'atmosphère, et dans ce domaine, Harde ne dit pas autre chose que le reste des scientifiques du climat, il reconnait bien que l'effet de serre radiatif existe. (Comment pourrait-il en être autrement puisque celui-ci est observable dans l'infra-rouge, on trouve dans tous les manuels de physique de l'atmosphère des spectres du rayonnement de l'atmosphère vers la Terre qui montrent clairement le rayonnement des GES !) Les bases scientifiques présentées au début de l'article me semblent solides, elles reposent en bonne partie sur un article d'Einstein publié en 1916 où il définit les coefficients qui portent son nom et qui caractérisent les phénomènes d'émission spontanée, d'émission et d'absorption induites. (La compréhension de ces phénomènes débouchera sur la technologie des lasers.) Cet article permet donc de comprendre ce qu'il se passe au niveau moléculaire avec le CO2. Il y a bien des milliards de collisions par seconde à la pression atmosphérique, et environ une fois sur mille, ces collisions sont inélastiques ou superélastiques, c'est à dire qu'il y a conversion d'énergie quantique en énergie thermique ou l'inverse. Mais contrairement à ce que semble affirmer Geuskens, cela n'empêche pas au rayonnement de se propager, sans même faire intervenir la convection. Pour qlq un qui expert dans ce domaine, ce que je reconte doit d'ailleurs paraître trivial, étant donné qu'il existe des software comme MODTRAN qui permettent de modéliser ce transfert radiatif, et qui sont utilisés pour faire des prévisions météo.

    RépondreSupprimer
  46. Je suis d'accord avec Josh, l'augmentation de l'altitude d'émission est fondamentale car elle permet de comprendre pourquoi il y a moins d'infra-rouge rayonné vers l'espace (la puisance rayonnée diminue avec la puissance 4 de la température qui diminue avec l'altitude). Cependant, cette altitude d'émission est une notion théorique, c'est pourquoi on l'appelle altitude effective d'émission. En réalité, le rayonnement est "libéré" sur une plage d'altitudes, comme on peut le voir sur un spectre OLR. Cette altitude d'émission est une simplification sans laquelle il est difficile d'aborder le phénomène.

    RépondreSupprimer
  47. @Unknown : L'altitude d'émission doit forcément augmenter avec la concentration en CO2, car la température de la troposphère augmente. Par définition, l'altitude d'émission est liée à une température précise, celle pour laquelle la puissance rayonnée est égale à l'énergie reçue du soleil moins l'albedo. Comme la puissance du soleil varie très peu, si la température de la tropo augmente, il faut monter plus haut pour retrouver la température qui correspond à l'altitude d'émission.

    RépondreSupprimer
  48. @BenHur : Je ne pense pas que la convection joue un rôle important car les échelles de temps sont très différentes entre la convection et la radiation.

    RépondreSupprimer
  49. Enfin de la science, avec plein de belles équations; ça m'a l'air tout à fait correct, M. Harde a compris qu'il y avait du rayonnement créé par l'atmosphère et pas seulement par la terre, et que l'effet de serre correspond à la réduction de la fuite du rayonnement émis vers l'espace; en fait, il a l'air d'avoir compris pas mal de choses; bon maintenant il faut digérer tout ça, ça va prendre un peu de temps.

    Maintenant une question: à la tropopause, le CO2 est encore très absorbant (voir figure 2.1 du premier rapport du GIEC); est-on bien sur que l'altitude d'émission (la zone d'émission, vous avez raison) se situe dans la troposphère ? moi, je la verrais volontiers dans la stratosphère, ce qui changerait tout, non ? qu'en pensez-vous ?

    En tout cas, merci Stef; êtes-vous Normalien ou Polytechnicien ?

    RépondreSupprimer
    Réponses
    1. « M. Harde a compris qu'il y avait du rayonnement créé par l'atmosphère et pas seulement par la terre »

      Excusez-moi d'intervenir sur ce point, mais malgré mes grosses lacunes sur le sujet il me semble avoir compris que le rayonnement s'effectuait non seulement depuis le sol mais également dans l'atmosphère, Harde n'est donc pas du tout le seul à avoir compris cela, à mon avis il ne fait que répéter ce que d'autres bien avant lui avaient découvert (voir par exemple http://www.climat-en-questions.fr/reponse/fonctionnement-climat/bilan-radiatif-terre-par-robert-kandel-serge-planton)

      Supprimer
    2. L'altitude effective d'émission se trouve à environ 5km, donc au milieu de la troposphère. On la calcule on cherchant l'altitude correspondant à 255K (température d'un corps noir qui émet 240W/m2) avec le gradient adiabatique. On obtient les altitudes réelles d'émission grâce au spectre OLR. Il y a encore sûrement un peu de rayonnement dans la tropopause, mais l'essentiel a été rayonné avant. On peut trouver des profils détaillés de l'atmosphère calculés avec MODTRAN sur le site "Science of doom".
      Je ne suis ni normalien ni polytechnicien, j'ai un master en physique, et je travaille dans un labo belge de physique des hautes énergies, mais ma spécialité, ce sont les systèmes de calcul et de stockage de données pour les scientifiques.

      Supprimer
    3. @Géd Je ne suis pas sûr de comprendre pas le sens de cette remarque de Josh... Harde montre comment l'atmosphère transmet le rayonnement émis par la Terre, mais comme vous le dites, ce n'est pas nouveau, on sait cela depuis longtemps. Le grand mérite du papier de Harde est d'examiner les choses au niveau moléculaire. La derivation de l'équation de Schartzchild qui décrit le transfert radiatif, on peut la trouver dans Wikipedia. Donc oui, rien de nouveau sous le soleil.

      Supprimer
    4. @Stef
      Je pense que Josh n'a tout simplement pas assez bossé son sujet et qu'il s'est laissé embrouiller par Geuskens ; je dis ça bien que je ne pige pas grand chose là-dedans, mais c'est mon petit doigt qui me donne des indications;)

      Supprimer
    5. Au fait, Gavin Schmidt n'est pas particulièrement fan d'Hermann Harde… voir http://www.realclimate.org/index.php/archives/2017/02/something-harde-to-believe/

      Personnellement je n'ai évidemment aucun avis sur la question, je fais seulement remarquer que Harde est contesté.

      Supprimer
    6. Et Gavin Schmidt n'est pas le seul, voici les avis de l'astrophysicien Ken Rice : https://andthentheresphysics.wordpress.com/2017/10/03/a-harde-response/ et https://andthentheresphysics.wordpress.com/2017/02/26/oh-no-not-again/

      Supprimer
    7. @Géd : au sujet de Harde :
      Il faut effectivement se méfier de Harde car il essaie, comme Geuskens, d'embrouiller les gens, mais avec beaucoup plus de sophistication. Gavin Schmidt et d'autres ont vivement critiqué ses papiers sur le CO2 car Harde confond le temps de résidence moyen du CO2 dans l'atmopshère avec le temps de résilience de l'atmosphère. Le papier que j'ai mentionné ici est aussi critiqué car, dans sa section 5, Harde explique que ses calculs conduisent à une sensibilité climatique bcp plus faible que celle du GIEC. Les gens qui se sont penchés sur ces calculs expliquent en gros que la modélisation de l'atmosphère utilisée par Harde est trop simple, et que cela explique pourquoi Harde obtient une sensibilité climatique 3 fois plus faible que celle du GIEC (sans les rétro-actions). Voici la critique : http://rabett.blogspot.com/2011/03/toy-model.html?m=1
      Cela étant dit, les bases qui sont exposées dans les sections 1 à 4 sont correctes, elles ne racontent pas autre chose que de ce qu'on peut trouver dans la littérature spécialisée (Petty, Goody & Yung,...), sauf que ces bouquins sont écrits pour des experts.

      Supprimer
    8. « les bases qui sont exposées dans les sections 1 à 4 sont correctes »

      C'est bien ça le hic, c'est la spécialité apparemment des climatosceptiques, ils prennent des choses justes, les mélangent et en sortent un peu n'importe quoi, d'où l'expression « c'est même pas faux » qui se réfère aux parties correctes ; au final on obtient une mélasse de laquelle il doit être très difficile voire impossible pour un profane de démêler le vrai du faux.

      Supprimer
  50. Quant aux articles de Geuskens dans Science-Climat-Energie, puisqu'il en a été bcp question, il m'est difficile de dire ce que j'en pense tout en restant poli.  C'est de la merde, et je pèse mes mots. Ce ne sont pas juste des coquilles qu'on y trouve, mais des perles de non-sens physique qui révèlent chacune une des nombreuses facettes de son ignorance et son incompréhension des phénomènes qui prévalent dans l'atmosphère. Quelques morceaux choisis :

    * " Une fraction des molécules de CO2 excitées peut alors se désactiver avec émission d’un rayonnement avant qu’une collision inélastique ne se produise avec les molécules environnantes. Ce n’est pas le cas dans les basses couches atmosphériques où l’émission d’un rayonnement de fluorescence du CO2 n’a jamais été observée."
    Dans ce court extrait, on découvre que Geuskens n'a jamais entendu parler du "collision broadening". Il pense donc que fluorescence et collisions sont des phénomènes concurrents. La dernière phrase révèle qu'il n'a jamais vu de spectre du DLR.

    * " Le rayonnement émis par un corps froid ne peut donc pas être absorbé par un corps chaud."
    Trouver cette ineptie dans un article censé nous enseigner le bon usage de la loi de Stefan-Boltzmann, c'est un comble.

    Une dernière perle pour la route, au sujet de Stefan-Boltzmann :
    * "Elle ne peut donc s’appliquer à la Terre où la convection de l’air et l’évaporation de l’eau des océans contribuent de manière prépondérante au bilan énergétique."
    La convection et l'évaporation ne contribuent pas au bilan énergétique de la Terre, leur seul effet est de redistribuer l'énergie au sein de l'atmosphère.

    Un spécialiste des polymères (un des core business de la pétro-chimie) à la retraite qui se mêle d'écrire des articles sur le climat, alors qu'il n'a jamais rien publié dans ce domaine, et qui prétend faire passer pour des demeurés l'ensemble des physiciens de l'atmosphère, ça devrait normalement susciter la méfiance. Mais apparemment, non.

    RépondreSupprimer
  51. Pour Stef

    Il y a beaucoup de gens qui n'ont pas compris qu'en plus du rayonnement IR émis par la terre, il y a aussi un rayonnement émis par l'atmosphère (M. Geuskens par exemple); et il y a aussi beaucoup de gens qui n'ont pas compris que l'effet de serre résultait de l'émission de rayonnement IR dans l'espace (M. Geuskens et beaucoup d'autres); je sais que ça n'est pas nouveau, mais je suis content de voir que M. Harde fait partie des gens qui ont compris ça.

    La preuve s'il en était besoin que je ne me suis pas du tout laissé embrouiller par Geuskens comme le pense Ged.

    L'altitude d'émission n'est pas la même pour la vapeur d'eau et pour le CO2; pour la vapeur d'eau, elle est bien de 5Km, mais elle est plus élevée pour le CO2; la figure 2.1 du premier rapport du GIEC montre qu'à la tropopause, le CO2 est encore 100 % absorbant autour de 15 microns:

    https://www.ipcc.ch/site/assets/uploads/2018/03/ipcc_far_wg_I_full_report.pdf

    Donc une partie non négligeable de la zone d'émission devrait se trouver dans la stratosphère ?

    RépondreSupprimer
    Réponses
    1. « je ne me suis pas du tout laissé embrouiller par Geuskens comme le pense Ged »

      Ah bon ?

      Et qui a écrit « Le Professeur Geuskens est donc tout à fait dans son domaine de compétence pour traiter ce sujet, et manifestement il touche sa bille en la matière » ?

      A part cela vous ne vous êtes pas fait embrouiller par Geuskens.

      Par ailleurs pourquoi nous citer sans cesse le tout premier rapport du GIEC ? Voici le dernier en date : https://www.ipcc.ch/site/assets/uploads/2018/02/WG1AR5_all_final.pdf

      Supprimer
  52. Vous avez raison, j'ai mis à peu près deux mois avant de comprendre le mécanisme par lequel le CO2 influait sur le climat; vous citez mes propos du 7 Octobre, mais vous pourriez aussi citer ceux du 10 Octobre:

    "Oui, c'est vrai, c'est ce que j'essaye d'expliquer dans tous mes messages: le rayonnement infrarouge émis par la terre est entièrement absorbé par le CO2 dans les cent premiers métres; ce n'est pas lui qui crée l'effet de serre, c'est le rayonnement crée dans l'atmosphère par excitation des molécules de CO2 avec les molécules d'air environnant; une part de ce rayonnement est émis vers l'espace; c'est une perte d'énergie pour l'atmosphère, qui se refroidit; une augmentation de la teneur en CO2 absorbe un peu de ce rayonnement, donc réduit la fuite de rayonnement vers l'espace , ce qui réchauffe l'atmosphère; c'est ça l'effet de serre décrit par le GIEC; M. Geuskens n'a pris en compte que le rayonnement émis par la terre; M. Dufresne aussi d'ailleurs; ils ont tort tous les deux, et le GIEC a raison."

    Voilà, MM. Geuskens et Dufresne ont tort tous les deux, et le GIEC a raison.

    J'ai finalement réalisé après deux mois d'errance que M. Geuskens n'avait pas compris le mécanisme de l'effet de serre; en fait, j'ai beaucoup appris en participant à votre blog, et je vous en suis très reconnaissant.

    RépondreSupprimer
    Réponses
    1. « mais vous pourriez aussi citer ceux du 10 Octobre »

      J'aurais pu, mais il n'en reste pas moins que Geuskens vous a induit dès le début en erreur et vous a fait perdre du temps car vous lui avez fait confiance, c'est cela qui importe.

      Quant à dire que Dufresne aurait tort contre ce que rapporte de GIEC j'ai quelques doutes, peut-être que vous n'avez pas bien compris ses explications ou qu'il s'est mal exprimé...

      Supprimer
  53. Pour ce qui est des rapports du GIEC, la figure 2.1 qui était dans le premier rapport n'a pas été reprise dans les rapports suivants; je suis donc obligé de faire référence à ce premier rapport si je veux parler de l'absorption du CO2 à la tropopause.

    RépondreSupprimer
    Réponses
    1. Dans le dernier rapport du GIEC il y a une partie très technique qui est au-dessus de mes compétences, la réponse à vos questions ne se trouve-t-elle pas dedans même sans le schéma du premier rapport ?

      Supprimer
  54. Je rappelle que je cherche à comprendre par quels mécanismes le CO2 influe sur le réchauffement climatique, avant prise en compte d'éventuelles rétroactions.

    Il faut pour cela connaître l'altitude d'émission à laquelle le CO2 se désactive par rayonnement plus que par collision, et comment cette altitude varie avec la concentration en CO2; la méthode décrite par Stef est valable pour l'atmosphère dans son ensemble, mais ne permet pas de préciser le rôle du CO2 dans cet ensemble.

    Il faut donc étudier les profils détaillés de l'atmosphère calculés avec MODTRAN; je suis allé sur le site "Science of doom" conseillé par Jef, mais je ne les ai pas trouvés; pouvez-vous m'aider ?

    L'énergie rayonnée dans la bande de 14 à 16 microns est faible du fait de l'absorption par le CO2 ; l'énergie rayonnée autour de 13 et 17 microns est plus importante, mais l'absorption par le CO2 y est trés faible; je cite le paragraphe 2.2.2 du premier rapport du GIEC:

    "The natural quantities of carbon dioxide are so large that the atmosphere is very opaque
    over short distances at the centre of its 15 microns band. At this wavelength the radiation reaching the tropopause, from both above and below, comes from regions at temperatures
    little different to the tropopause itself. The net flux is thus close to zero. The addition of a small amount of gas capable of absorbing at this wavelength has negligible effect on the net flux at the tropopause The effect of added carbon dioxide molecules is, however, significant at the Wavelength edges of the 15 microns band, and in particular around 13. 7 and 16 microns."

    Ce commentaire et la figure correspondante ne sont pas repris dans les quatre rapports ultérieurs du GIEC, c'est pourquoi je ne peux citer que ce premier rapport.

    Si l'énergie est faible, la variation d'énergie est nécessairement faible et devrait donc conduire à une sensibilité climatique faible.

    Comme dit M. Dufresne, ce problème est "crucial"; les calculs correspondants devraient figurer dans les rapports du GIEC, mais je n'y ai trouvé que des formules du genre k. ln (C/Co); ont-ils été publiés quelque part ?

    Encore une fois, je ne vois pas comment on peut quantifier le rôle du CO2 dans le réchauffement climatique sans effectuer ces calculs, avant prise en compte des rétroactions.

    RépondreSupprimer
    Réponses
    1. @Josh :
      Ici : https://scienceofdoom.com/2013/01/13/visualizing-atmospheric-radiation-part-seven-co2-increases/

      Supprimer
  55. Merci Stef; cet article est très intéressant mais pas très facile à comprendre.

    Néanmoins, en prenant le résultat final de 4,3 W x m-1 d'augmentation d'absorption pour un doublement de la concentration en CO2, on trouve une sensibilité climatique (dérivée de la formule de Boltzmann, voir mon message du 7 Octobre) de:

    288 /4 x 4,3 / 238,5 = 1,3 K

    La température augmente de 1,3 K quand la teneur en CO2 double; c'est dans la fourchette basse du GIEC et dans la fourchette haute des sceptiques.

    Aujourd'hui, nous sommes à 400 ppm de CO2, il faut en rajouter 400 ppm pour doubler la concentration, qui augmente de 2 ppm par an; il faudra donc 400 /2 = 200 ans pour doubler la concentration en CO2.

    L'augmentation de température due à l'augmentation de C02 est donc de 1,3 / 200 = 6,3 millième de degré par an (j'avais calculé un centième de degré par an dans mon message du 7 Octobre).

    En 2050, dans 30 ans, du fait de l'augmentation de la teneur en CO2 dans l'atmosphère, la température aura augmenté de 0,0065 x 30 = 0,2 K, et en 2100, dans 80 ans : 0,0065 x 80 = 0,5 K.

    La question qui se pose maintenant est de savoir quelles preuves avons-nous pour considérer que de si faibles augmentations de température sont capables de générer les rétroactions positives incluses dans les calculs du GIEC.

    RépondreSupprimer
    Réponses
    1. Et vous ne semblez pas comprendre que 0,17 degré par décénnie (c'est l'augmentation mesurée), c'est énorme. Il a fallu des milliers d'années pour sortir de la dernière glaciation, càd gagner 4-5 degrés. L'augmentation que nous connaissons maintenant n'a jamais été observée, en ampleur et en rapidité, au cours des 800.000 dernières années.

      Supprimer
    2. "La question qui se pose maintenant est de savoir..." Vous êtes serieux ??? Nous sommes actuellement en train de vivre les effets de +1 degré : vagues de chaleur, fonte des glaciers, du permafrost, etc. etc. Et vous vous demandez comment cela peut avoir un effet ? (Re)lisez les papiers de Lacis et Schmidt, le climat mondial est une machine très sensible. Vous avez déjà entendu parler des cycles de Milankovitch ? Connaissez vous la valeur des forçages dus à ces cycles ? On est dans les mêmes ordres de grandeur que le forçage du au CO2.

      Supprimer
    3. Je rajouterai que pour retrouver une telle concentration en CO2 il faut remonter à plus de trois millions d'années (voir http://sogeco31.blogspot.com/2019/05/415-ppm-de-co-mais-ce-nest-pas-une.html) ; on peut même remonter jusqu'à 15 millions d'années en arrière pour trouver l'équivalent d'aujourd'hui !

      Et comme vous le faites remarquer à Josh cela s'est traduit par des variations de températures d'environ 5 degrés étalées sur des milliers d'années, rien à voir donc avec ce qui se passe depuis un siècle ou deux (et surtout depuis une quarantaine d'années) ; si en 2100 nous avons +5°C (ce qui est relativement probable) nous aurons réussi l'exploit de faire en un siècle ce que la nature faisait en plusieurs milliers d'années, nous pourrons en être fiers !

      Supprimer
  56. Vous oubliez les rétro-actions positives : augmentation de la vapeur d'eau, diminution de l'albedo, diminution de l'efficacité des puits naturels, libération de GES (méthane) due à la fonte du permafrost, etc.

    RépondreSupprimer
  57. Bonjour à tous et merci pour vos commentaires ; je ne conteste pas le réchauffement climatique, mais je demande des preuves que c’est bien le CO2 qui est responsable des rétroactions positives incluses dans les calculs publiés par le GIEC.

    En l’absence de telles preuves, je vais donner une preuve du contraire : d’après le GIEC (glossaire du 5ème rapport), l’effet de serre est dû à l’augmentation de l’altitude d’émission avec l’augmentation de la concentration en gaz à effet de serre ; ce phénomène est bien expliqué dans l’article de M. Dufresne : « L’effet de serre atmosphérique : plus subtil qu’on ne le croit ! »

    En dessous de l’altitude d’émission, la concentration en molécules de gaz à effet de serre est élevée, et tout le rayonnement infrarouge est absorbé ; avec l’augmentation de l’ altitude, la concentration en GES décroit et à partir d’une certaine altitude dite altitude d’émission, une partie du rayonnement IR n’est plus absorbé et est rayonné vers l’espace, ce qui refroidit la planète (terre + atmosphère), et compense ainsi le rayonnement incident reçu du soleil.

    Lorsque la concentration en GES augmente, l’altitude d’émission augmente aussi (cela ne me parait pas évident, mais c’est ce qu’expliquent le GIEC et M. Dufresne) ; la température diminue avec l’altitude, ce qui provoque une baisse du rayonnement des GES, c'est-à-dire une baisse du rayonnement émis vers l’espace, donc une diminution du refroidissement de la planète, c'est-à-dire un réchauffement.

    Voilà le mécanisme de l’effet de serre tel que décrit par le GIEC, et expliqué en détail par M. Dufresne.

    Ce mécanisme est certainement valable pour la vapeur d’eau, dont l’altitude d’émission se situe à environ 5 Km, au milieu de la troposphère comme expliqué par Jef ; j’ai consulté le site qu’il m’a conseillé, "Science of doom", mais pas de trace d’altitude d’émission ni de spectre du rayonnement IR à la tropopause.

    Je m’en remets donc au paragraphe 2.2.2 du premier rapport du GIEC, que j’ai cité à de nombreuses reprises, en vain semble-t-il ; le texte de ce paragraphe et sa figure 2.1 montrent qu’à la tropopause, le CO2 est toujours 100 % absorbant, ce qui est confirmé par d’autres publications, et n’est pas démenti par « Science of Doom ».

    L’altitude d’émission du CO2 se situe donc dans la stratosphère, où la température augmente avec l’altitude ; une augmentation de la concentration en CO2, en provoquant une augmentation de l’altitude d’émission (d’après le GIEC et M. Dufresne) va donc entraîner une augmentation de la température du CO2, donc une augmentation du rayonnement IR émis vers l’espace, c'est-à-dire un refroidissement de la planète.

    Je n’ai pas cité une seule ligne d’un article des climato-sceptiques, seulement le premier et le cinquième rapport du GIEC, et la conclusion s’impose : d’après ces rapports, l’augmentation de la concentration en CO2 provoque un refroidissement de la planète.

    Je lirai vos commentaires avec beaucoup d’intérêt.

    RépondreSupprimer
  58. Bien qu'il soit assez difficile de s'y retrouver, les spectres d'absorption IR calculés dans "Science of Doom" montrent qu'a 16,9 Km d'altitude, le CO2 reste absorbant à 90 %, et à 85 % au sommet de l'atmosphère (TOA), ce qui confirme que l'altitude d'émission du CO2 se trouve bien dans la stratosphère.

    RépondreSupprimer
  59. J'aimerais bien avoir votre avis, ainsi que celui de VB et Stef sur ce qui précède.

    D'avance, merci

    RépondreSupprimer
  60. @Josh
    L'altitude du sommet de la troposphère varie en fonction de la latitude, entre 9km et 17km si ma mémoire est bonne. Cette altitude est en fait liée à la température au sol, et on peut aisément la calculer en égalant l'énergie cinétique (3/2 kT) et l'énergie potentielle (m g h, où h est l'altitude max.).
    Le graphique 2.1 que vous mentionnez dans le premier rapport du GIEC montre le flux net sortant de la tropopause, c'est d'ailleurs indiqué dans la légende. Effectivement, à 15um, c'est nul, mais ça augmente rapidement dès que l'on s'écarte de 15 um. Néanmoins, si vous consultez des graphiques du OLR, c'est à dire du flux d'infra-rouge mesuré depuis l'espace, vous verrez qu'on ne tombe pas à 0 à 15 um. Il y a effectivement absorption et émission de photons à des longueurs d'ondes proches de 15 um dans la stratosphère, mais cela reste marginal et ne concerne qu'une partie très étroite du spectre. Le processus dominant dans la stratosphère est l'absorption d'UV par l'ozone.
    Par ailleurs, vous ne pouvez extrapoler le raisonnement de Dufresne dans la stratosphère car celle-ci n'a pas de gradient thermique adiabatique.
    En conclusion partielle : Affirmer, comme vous le faites dans votre dernier commentaire, que l'altitude d'émission du CO2 est dans la stratosphère est totalement abusif (cherry-picking) et est contredit par les différents spectres dont nous avons discuté.
    De manière plus générale, soutenir que le CO2 contribue à refroidir la planète est en contradiction frontale avec les observations : le CO2 augmente et la troposphère se réchauffe. Le fait que dans le même temps la stratosphère se refroidit est une signature d'un effet de serre radiatif accru. Et plusieurs études apportent des observations directes de cet accroissement de l'effet de serre : https://www.nature.com/articles/nature14240 ou encore https://www.nature.com/articles/35066553

    RépondreSupprimer
  61. @Josh
    Et il y a aussi le papier de Benjamin Santer(https://www.nature.com/articles/s41558-019-0424-x), publié en début d'année, dont voici un extrait :
    "Because of this confluence in scientific understanding, we can now answer the following question: when did a human-caused tropospheric warming signal first emerge from the background noise of natural climate variability? We addressed this question by applying a fingerprint method related to Hasselmann’s approach (see online Methods). An anthropogenic fingerprint of tropospheric warming is identifiable with high statistical confidence in all currently available satellite datasets (Figure 1). In two out of three datasets, fingerprint detection at a 5-sigma threshold – the gold standard for discoveries in particle physics– occurs no later than 2005, only 27 years after the 1979 start of the satellite measurements.Humanity cannot afford to ignore such clear signals."

    RépondreSupprimer